Sei sulla pagina 1di 20

INTEGRATED REVIEW 2: Advanced Financial Accounting and Reporting (AFAR)

#1 | Partnership Accounting: Nature, Formation, Operations, Dissolutions, Liquidation B. P 18,000


C. P 30,000
D. P 0
1. The JPB partnership reported net income of P160,000 for the year ended December 31, 4. How much cash would Janis receive upon final liquidation, assuming no prior cash
20x4. According to the partnership agreement, partnership profits and losses are to be distribution had been made to the partners.
distributed as follows: A. P 135,000
B. P 145,000
J P B C. P 100,000
Salaries P 50,000 P60,000 P30,000 D. P 0
Bonus on net income 10% 5% 10% (PRTC, May 2018)
Remainder (if positive) 60% 30% 10%
Remainder (if negative) 30% 40% 30% 5. PP contributed P24,000 and CC contributed P48,000 to form a partnership, and they agreed
to share profits in the ratio of their original capital contributions. During the first year of
How should partnership net income for 20x4 be allocated to J, P, and B? operations, they made a profit of P16,290; PP withdrew P5,050 and CC P8,000. At the start
J P B of the following year, they agreed to admit GG into the partnership. He was to receive a
A. P96,000 P48,000 P16,000 one-fourth interest in the capital and profits upon payment of P30,000 to PP and CC, whose
B. P58,000 P64,000 P38,000 capital accounts were to be reduced by transfers to GG’s capital account of amounts
C. P60,000 P60,000 P40,000 sufficient to bring them back to their original capital ratio.
D. P66,000 P68,000 P46,000
How should the P30,000 paid by GG be divided between PP and CC?
(RESA, May 2018) A. PP, P 9,825; CC, P 20,175
2. DO is admitted into the partnership of RE and MI by investing cash equivalent to ¼ of their B. PP, P 15,000; CC, P 15,000
capital. Which of the following is true after the admission of DO? C. PP, P 10,000; CC, P 20,000
A. Assets of the partnership will increase D. PP, P 9,300; CC, P 20,700
B. Total partners’ equity remain the same (Dayag, 2015)
C. RE and MI capital decreased by ¼
D. Assets of the partnership will remain the same 6. Scott, Joe, and Ed are liquidating their partnership. At the date the liquidation begins Scott,
Joe, and Ed have capital account balances of P162,000, P192,500, and P215,000,
(RESA, May 2018) respectively and the partners share profits and losses 40%, 35%, and 25%, respectively. In
For numbers 3-4, refer to the problem below: addition, the partnership has a P36,000 Notes Payable to Scott and a P20,000 Notes
The following condensed balance sheet is presented at February 18, 2018 for the Receivable from Ed. When the liquidation begins, what is the loss absorption power with
partnership of Dana and Janis, who share profits and losses in ratio of 60:40, respectfully. respect to Joe?
Cash P150,000 Accounts payable P120,000 A. P192, 500
Non-cash assets 300,000 Dana, Capital 195,000 B. P 67,375
Dana, Loan 20,000 Janis, Capital 155,000 C. P550,000
D. P770,000
The non-cash assets realized P250,000 in actual liquidation (Dayag, 2015)
3. How much would Dana receive if cash is distributed to the partners just before the start of 7. Which of the following is not considered a legitimate expense of a partnership?
actual liquidation? A. Interest paid to partners based on the amount of invested capital
A. P 5,000 B. Depreciation on assets contributed to the partnership by partners

C. Salaries for management hired to run the business A. 2,3,4,1


D. Supplies used in the partners’ offices B. 2,3,1,4
(Punzalan, 2014) C. 3,2,1,4
8. In the AA-BB partnership, AA and BB had a capital ratio of 3:1 and a profit and loss D. 3,2,4,1
ratio of 2:1 respectively. The bonus method was used to record CC’s admittance as a new (Punzalan, 2016)
partner. What ratio would be used to allocate, to AA and BB, the excess of CC’s
contribution over the amount credited to CC’s capital account? 13. It is the change in the relation of the partners caused by any partner ceasing to be associated
A. AA and BB’s new relative ratio. in the carrying on of the business.
B. AA and BB’s new relative profit and loss ratio. A. Dissolution
C. AA and BB’s old capital ratio. B. Liquidation
D. AA and BB’s old profit and loss ratio. C. Incorporation
D. Break-up
(Dayag 2013) (Millan, 2017)
9. The following is the priority sequence in which liquidation proceeds will be distributed 14. On January 1, 2016, Atta and Boy agreed to form a partnership contributing their
for a partnership: respective assets and equities subject to adjustment. On that date, the following were
A. Partnership drawings, partnership liabilities, partnership loans, partnership capital provided:
balances
B. Partnership liabilities, partnership loans, partnership capital balances. Atta Boy
C. Partnership liabilities, partnership loans, partnership drawings, partnership capital Cash 28,000 62,000
balances. Accounts receivable 200,000 600,000
D. Partnership liabilities, partnership capital balances, partnership loans Inventories 120,000 200,000
Land 600,000
(Punzalan, 2014) Building 500,000
10. Partnership capital and drawings accounts are similar to the corporate Furniture and Fixtures 50,000 35,000
A. Paid in capital, retained earnings, and dividends accounts. Intangible assets 2,000 3,000
B. Retained earnings accounts Accounts Payable 180,000 250,000
C. Paid in capital and retained earnings accounts Other liabilities 200,000 350,000
D. Preferred and common stock accounts. Capital 620,000 800,000
(Punzalan, 2014)

11. An advantage of the partnership as a form of business organization would be


A. Partners do not pay income taxes on their share in partnership income. The ff adjustments were agreed upon:
B. A partnership is bound by the act of the partners. A. Accounts receivable of P 20,000 and P 40,000 are uncollectible in A’s and B’s
C. A partnership is created by mere agreements of the partners. respective books.
D. A partnership may be terminated by the death or withdrawal of a partner. B. Inventories of P 6,000 and P 7,000 are worthless in A’s and B’s respective books
(Punzalan, 2016) C. Intangible assets are to be written off in both books.
What will be the capital balances of the partners after adjustments?
12. In the liquidation of a partnership it is necessary to (1) distribute cash to the partners; (2)
sell non-cash assets; (3) allocate any gain or loss on realization to the partners; and (4) pay Atta Boy
liabilities. These steps should be performed in the following order A. 592,000 750,00
B. 600,000 700,00 17. Eddy decided to retire from the partnership by mutual agreement is to be paid P 180,000
C. 592,000 756,300 out of partnership funds for his interest. Total goodwill implicit in the agreement is to be
D. 600,000 750,000 recorded. After Eddy’s retirement, what will be capital balances of the other partners?
(Punzalan, 2016) Fox Grimm
A. 84,000 56,000
15. Partner Ae first contributed P50,000 of capital into existing partnership on March 1, 2016. B. 102,000 68,000
On June 1, 2016, said partner contributed another P20,000. On September 1, 2016, he C. 108,000 72,000
withdrew P15,000 from the partnership. Withdrawal in excess of P10,000 are charged to D. 120,000 80,000
partner’s capital accounts. What is the annual weighted average capital balance of Partner (Punzalan, 2016)
Ae?
A. 32,500 18. Assume instead that Eddy remains in the partnership and that Hamm is admitted as a new
B. 51,667 partner with 25% interest in the capital of the new partnership for a cash payment
C. 60,000 P140,000. Total goodwill implicit in the transaction is to be recorded. Immediately after
D. 48,333 admission of Hamm, Eddy’s capital account balance should be
(Punzalan, 2016)
A. 280,000
16. Maxwell is trying to decide whether to accept a salary of P 40,000 or salary of P 25,000 B. 210,000
plus a bonus of 10% of net income after salaries and bonus as a means of allocating profit C. 160,000
among partners. Salaries traceable to the other partners are estimated to be P 100,000. D. 140,000
What amount of income would be necessary so that Maxwell would consider choices to (Punzalan, 2016)
be equal For numbers 9 and 10 refer to the problem below:
A. 165,000 The ABC Partnership has assets with book value of P240,000 and a market value of
B. 290,000 P195,000, outside liabilities of P70,000, loans payable to Partner Able of P20,000, and
C. 265,000 capital balances for Partners Able, Baker and Chapman of P70,000, P30,000 and P50,000,
D. 305,000 respectively. The partners share profits and losses equally.
(Punzalan, 2016)
19. How would the first P100,000 of available assets be distributed?
For numbers 17 and 18 refer to the problem below: A. P70,000 to outside liabilities, P20,000 to able and balances equally among partners
On June 30, 2016, the condensed balance sheet for the partnership of Eddy, Fox, and B. P70,000 to outside liabilities, and P30,000 to Able
Grimm together with their respective profit and loss sharing percentage, were as follows: C. P70,000 to outside liabilities, P25,000 to Able, and P5,000 to Chapman
D. P40,000 to Able, P20,000 to Chapman, and the balance equally among partners
Assets, net of liabilities P 320,000 (Punzalan, 2017)

Eddy, Capital (50%) P 160,000 20. If all outside creditors and loans to partners had been paid. How would the balance of the
Fox, Capital (30%) 96,000 assets be distributed assuming Chapman had already received assets with a value of
Grimm, Capital (20%) 64,000 P30,000?
Total Capital P 320,000 A. Each of the partners would receive P30,000
B. Each of the partners would receive P40,000
C. Able: P70,000; Baker: P30,000; Chapman: P20,000
D. Able: P55,000; Baker: P15,000; Chapman: P5,000

(Punzalan, 2017) to Grey and 40% to Redd. To form the partnership, Grey initially contributed assets costing
P30,000 with a fair value of P60,000 on January 2, 2016, and Redd contributed P20,000 cash.
Drawings by the partners during 2016 totaled P3,000 by Grey and P9,000 by Redd. The partnership
21. If a partner’s capital balance is credited for an amount greater than or less than the fair
net income in 2016 was P25,000.
value of his net contribution, the excess or deficiency is called a
(Punzalan 2018)
A. Bonus
B. Goodwill
25. Under the goodwill method, what is Redd’s initial capital balance in the partnership?
C. Discount
A. 20,000
D. Premium
B. 25,000
(Millan, 2016)
C. 40,000
D. 60,000
22. Before allocation of loss, which of the following items are allocated first?
A. Salaries
26. Under the bonus method, what is the amount of bonus?
B. Bonuses to partners
A. 20,000 bonus to Grey
C. Interest on the capital of an industrial partner
B. 20,000 bonus to Redd
D. All of these
C. 40,000 bonus to Grey
(Millan, 2016)
D. 40,000 bonus to Redd
23. After the admission of a new partner, the total partnership capital increased by the fair
27. If a partnership has net income of P44,000 and Partner X is to be allocated bonus of 10%
value of the new partner’s net contributions to the partnership. The admission was
of income after the bonus, what is the amount of bonus Partner X will receive?
accounted for
A. 3,000
A. Under the goodwill method
B. 3,300
B. Under the bonus method
C. 4,000
C. As a purchase of interest
D. 4,400
D. As an investment in the partnership
(Punzalan, 2018)
(Millan, 2016)

24. On May 1, 2016, Cobb and Mott formed a partnership and agreed to share profits and losses
28. A partnership has the following accounting amounts:
in the ratio of 3:7, respectively. Cobb contributed a parcel of land that cost him P10,000.
Sales P 700,000
Mott contributed P40,000 cash. The land was sold for P18,000 on May 1, 2016,
Cost of goods sold 400,000
immediately after formation of the partnership. What amount should be recorded in
Operating expenses 100,000
Cobbs’s capital account on formation of the partnership?
Salary allocations to partners 130,000
A. 18,000
Interest paid to banks 20,000
B. 17,400
Partners' drawings 80,000
C. 15,000
D. 10,000 What is the partnership net income (loss)?
(Punzalan, 2018) A. 200,000
B. 180,000
For numbers 5 to 6: C. 50,000
The Grey and Redd Partnership was formed on January 2, 2016. Under the partnership agreement, D. (30,000)
each partner has an equal initial capital balance. Partnership net income or loss is allocated 60% (Punzalan, 2018)
Solution: 32. During 2016, Young and Zinc maintained average capital balances in their partnership of
160000 and 100000, respectively. The partners receive 10% interest on average capital
29. Ranken purchases 50% of Lark’s capital interest in the K and L partnership for P22,000. If balances and residual profit or loss is divided equally. Partnership profit before interest was
the capital balances of Kim and Lark are P40,000 and P30,000, respectively, Ranken’s 4000. By what amount should Zinc’s capital account change for the year?
capital balance following the purchase is A. 11000 decrease
A. 22,000 B. 2000 increase
B. 35,000 C. 1000 decrease
C. 20,000 D. 12000 increase
D. 15,000 (Punzalan, 2018)
(Punzalan, 2018)

30. The following condensed balance sheet is presented for the partnership of Smith and Jones, Mitz, Marc and Mart are partners sharing profits in the ratio of 5:3:2, respectively. As of
who share profits and losses in the ratio of 60:40, respectively: December 31, 2016, their capital balances were 95,000 for Mitz, 80000 for Marc & 60000 for
Other assets P 450,000 Mart. On Jan 1, 2017, the partners admitted Vince as a new partner and according to their
Smith, loan 20,000 agreement, Vince will contribute 80000 in cash to the partnership and also pay 10000 for 15%
P 470,000 of Marc’s share. Vince will be given a 20% share in profits, while the original partners’ share
will be approximately the same as before. After the admission of Vince, the total capital will
be 330000 and Vince’s Capital will be 70000
Accounts payable P120,000
Smith, capital 195,000
Jones, capital 155,000
33. The total amount of goodwill to the old partners, upon the admission of Vince would be:
P 470,000
A. 7000
B. 15000
The partners decided to liquidate the partnership. If the other assets are sold for P385,000, C. 22000
what amount of the available cash should be distributed to Smith? D. 37000
A. 136,000
B. 156,000
C. 159,000 34. . The balance of Marc’s Capital, after admission of Vince would be:
D. 195,000 A. 72600
(Punzalan, 2018) B. 74600
C. 79100
31. Flat and Iron partnership agreement provides for Flat to receive 20% bonus on profits D. 81100
before bonus. Remaining profits and losses are divided between Flat and Iron in the ratio (Punzalan, 2018)
2:3, respectiviely. Ehich partner has greater advantage when the partnership has a profit or
when it has a loss
A. Profit: Flat; Loss:Iron 35. As of Dec 31, the books of AME Partnership showed capital balances of A - 40,000; M
B. Profit:Flat; Loss: Flat 25,000; E-5,000. The partners’ profit or loss ratio is 3:2:1, respectively. The partners
C. Profit: Iron; Loss: Flat decided to dissolve and liquidate. They sold all the non-cash assets for 37,000 cash. After
D. Profit: Iron; Loss: Iron settlement of all liabilities amounting to 12,000, they still have 28,000 cash left for
(Punzalan, 2018) distribution. The loss on realization for distribution is

A. 40,000 B. The income summary account is credited in the entry to record the distribution of
B. 42,000 profits
C. 44,000 C. In the absence of any agreement, salary allowances to partners shall be provided
D. 45,000 when the operations yield losses
(Punzalan, 2018) D. Salary and interest allowances are reported in the statement of comprehensive
income as salaries and interest expense
36. In installment liquidation, which of the following statements is correct regarding the
partial settlement of the partners’ claims?
40. Partners C & K share profits and losses equally after each has been credited in all
circumstances with annual salary allowances of 15,000 & 12,000, respectively. Under this
A. The claims of the partners and outside creditors are partially settled in proportion
arrangement, C will benefit by 3,000 more than K in which of the following:
B. No distribution is made to the partners until after all non cash assets are realized
C. The carrying amount of unsold non cash assets is treated as loss
D. Estimates of future liquidation costs do not affect the distribution to the partners 1. Only if the partnership has earnings of 27,000 or more for the year
(Milan, 2016) 2. Only If the partnership does not incur a loss for the year
3. In all earnings or loss situation
37. Under the entity theory, a partnership is
4. Only if the partnership has earnings of at least 3000 for the year
(FT&C 11e)
A. Viewed as having its own existence apart from the partners
41. On June 30, 2015, the balance sheet of Western Marketing, a partnership, is summarized
B. Viewed through the eyes of the partnera
as follows:
C. A separate legal and tax entity
Sundry assets…………………………………………………………….P150,000
D. Unable to enter into contracts in its own name
West, Capital…………………………………………………………….…90,000
(FT&C)
Tern, Capital………………………………………………………………. 60,000
Wes and Tern share profit and losses at a 60:40 ratio, respectively. They agreed to take in
38. Which of the following statements is true concerning the treatment of salaries in Cuba as a new partner, who purchases 1/8 interest of West and Tern for P25,000. What is
partnership accounting? the amount of Cuba’s capital to be taken up in the partnership books if the book value
method is used?
A. The salary of a partner is treated in the same manner as salaries of corporate A. P12,500
employees B. P18,750
B. Partner salaries are equal to the annual partner draw C. P25,000
C. Partner salaries may be used to allocate profits and losses; they are not D. P31,250
considered expenses of the partnership (Dayag, 2015)
D. Partners salaries are directly closed to the capital account
(FT&C) 42. In the AD partnership, Allen’s capital is P140,000 and Daniel’s is P40,000 and they share
income in a 3:1 ratio, respectively. They decide to admit David to the partnership.

39. Which of the following is true? Allen and Daniel agree that some of the inventory is obsolete. The inventory account is
decreased before David is admitted. David invests P40,000 for a one-fifth interest. What is
A. A stipulation that excludes one or more partners from any share in the profits or the amount of inventory written down?
losses is valid
A. P4,000
B. P10,000 46. For financial accounting purposes, assets of an individual partner contributed to a
C. P15,000 partnership are recorded by the partner at
D. P20,000
(Dayag, 2015) A. Historical cost
B. Book value
C. Fair market value
43. In the AD partnership, Allen’s capital is P140,000 and Daniel’s is P40,000 and they share D. Lower of cost or market
income in a 3:1 ratio, respectively. They decide to admit David to the partnership. David (Dayag, 2015)
directly purchases one-fifth interest by paying Allen P34,000 and Daniel P10,000. The land
account is increased before David is admitted. By what amount is the land account 47. Which of the following interest component calculation bases is the least susceptible to
increased? manipulation when allocating profits and losses to partners?

A. P40,000 A. Beginning capital account balance


B. P36,000 B. Average of beginning and ending capital balance
C. P20,000 C. Weighted average capital account balance
D. P10,000 D. Ending capital balance
(Dayag, 2015) (Dayag, 2015)

44. RR and XX formed a partnership and agreed to divide initial capital equally, even though 48. In a partnership, interest on capital investment is accounted for as a(n)
RR contributed P25,000 and XX contributed P21,000 in identifiable assets. Under the
bonus method approach to adjust the capital accounts, XX’s unidentifiable assets should A. Return on investment
be debited for: B. Expense
C. Allocation of net income
A. P11,500 D. Reduction of capital
B. P4,000 (Dayag, 2015)
C. P2,000
D. 0 49. What is the underlying purpose of the interest on capital balances component of allocating
(Dayag, 2015) partnership profits and losses?

45. Partner A first contributed P50,000 of capital into an existing partnership on March 1, 2015. A. Compensate partners who contribute economic resources to the partnership
On June 1, 2015, the partner contributed another P20,000. On September 1, 2015, the B. Reward labor and expertise contributions
partner withdrew P15,000 from the partnership. Withdrawals in excess of P10,000 are C. Reward for special responsibilities taken
charged to the partner’s capital account. The annual weighted-average capital balance is D. None of the above
(Dayag, 2015)
A. P62,000
B. P51,667 50. What is the underlying purpose of the salary component of allocating partnership profits
C. P60,000 and losses?
D. P48,333
(Dayag, 2015) A. Compensate partners who contribute economic resources to the partnership

B. Reward labor and expertise contributions


C. Reward for special responsibilities taken The capital balances of the partners after Chic’s admission are:
D. None of the above A. Pol, P214,062.50; Loc, P104,687.50; and Chic, P106,250
(Dayag, 2015) B. Pol, P200,000.00; Loc, P100,000.00; and Chic, P125,000
C. Pol, P239,062.50; Loc, P 79,687.50; and Chic, P125,000
51. A, B, and C are partners in an accounting firm. Their capital account balances at year-end
D. Pol, P250,000.00; Loc, P125,000.00; and Chic, P100,000
were A P90,000; B P110,000; C P50,000. They share profits and losses on a 4:4:2 ratio,
(Guerrero, 2013)
after the following special terms:
• Partner C is to receive a bonus of 10% of net income after the bonus.
54. Michelle, an active partner in the Michelle-Esme partnership receives an annual bonus of
• Interests of 10% shall be paid on that portion of a partner’s capital in excess of 25% of the partnership income after deducting the bonus. For the year ended, December
P100,000
31, 2013, partnership income before the bonus amounted to P240,000. The bonus of
• Salaries of P10,000 and P12,000 shall be paid to partners A & C respectively. Michelle for the year 2013 is:
Assuming a net income of P44,000 for the year, the total profit share of Partner C was: A. P45,000
A. P7,800 B. P48,000
B. P16,800 C. P80,000
C. P19,400 D. P60,000
D. P19,800 (Guerrero, 2013)
(Dayag, 2015)
55. Rita, Sisa, and Tina are partners with the capital balances on June30, 2013 of P60,000,
52. X and Y are in partnership, sharing profits equally and preparing their accounts to 31 P60,000 and P40,000, respectively. Profits and losses are shared equally. Tina withdraws
December each year. On 1 July 2015, Z joined in the partnership, and from that date profits from the partnership. The partners agree that Tina is to take certain furniture at their
are shared X 40%, Y 40%, and Z 20%. secondhand value of P2,400 and cash for the balance of her interest. The furniture is carried
on the books as fully depreciated.
In the year ended 31 December 2015, profits were:
6 months to 31 June 2015 P200,000 The amount of cash to be paid to Tina and the capital balances of the remaining partners
6 months to 31 December 2015 P300,000 after the retirement of Tina are:
Cash Rita, capital Sisa, capital
It was agreed that X and Y only should bear equally the expense for a bad debt of P40,000 A. P40,000 P60,000 P60,000
written-off in the six months to 31 December 2015 in arriving at the P300,000 profit. B. P37,000 P61,200 P61,200
C. P38,400 P60,800 P60,800
Which of the following correctly states X’s profit share for the year? D. P42,800 P58,800 P58,800
A. P216,000 (Guerrero, 2013)
B. P200,000
C. P220,000 For question 55-56 refer to the problem below
D. P224,000 The AA, BB, CC Partnership was formed on January 2. 2019. The original cash
(Dayag, 2015) investments were as follows:
AA P 48,000
53. Pol and Loc are partners with capitals of P200,000 and P100,000 and sharing profits and BB P 72,000
losses 3:1 respectively. They agree to admit Chic as partner. Chic invests P125,000 for a CC P108,000
25% interest in the firm. Parties agree that the total firm capital after Chic’s admission is
to be P425,000.
According to the general partnership contract, the partners were to be remunerated as Annual interest on average capital account balance 10% 10%
follows: Remainder 60% 40%
a. Salaries of P72,000 for AA, P6,000 for BB, and P6,800 for CC.
b. Interest at 12% on the average capital account balances during the year. The annual salary was to be withdrawn by each partner in 12 monthly installments. During
c. Remainder divided 40% to AA, 30% to BB, and 30% for CC. fiscal year ended, February 28, 2020, DD and EE had net sales of P25,000, cost of goods
Income before partners’ salaries for the year ended December 31, 2019, was P46,040. AA sold of P140,000 and total operating expenses of P50,000 (excluding partners’ salaries and
invested an additional P12,000 in the partnership on July 1; CC withdrew P18,000 from interest on average capital account balances). Each partner made monthly cash drawings
the partnership on October 1, and, as authorized by the partnership contract, AA, BB, and in accordance with partnership contract.
CC each withdrew P375 monthly against their shares of net income for the year.
59. Determine the share of partner DD in the net income:
56. Determine the share of partner AA in the net income: A. P29,400
A. P18,416 B. P33,000
B. P17,616 C. P36,000
C. P13,080 D. P23,400
D. P5,880 (ReSA, 2018)
(ReSA, 2018)
60. The capital balance of each partner on March 1, 2020 should be:
57. Determine the capital balance of partner CC on December 31, 2019: A. DD P95,400; EE P138,600
A. P108,770 B. DD P66,000; EE P82,000
B. P104,270 C. DD P108,000; EE P147,000
C. P100,112 D. DD P99,000; EE P135,000
D. P99,312 (ReSA, 2018)
(ReSA, 2018)
61. Which of the following is not considered a legitimate expense of a partnership?
58. If the salaries to partners are to be recognized as operating expenses by the partnership, a. Supplies used in the partners’ office
what amount is the share of partner BB in the net income? b. Depreciation on assets contributed to the partnership by partners
A. P18,416 c. Salaries for management hired to run the business
B. P14,190 d. Interest paid to partners based on the amount of invested capital
C. P8,190
D. P7,812 Millan Textbook (2016)
(ReSA, 2018)
62. If the partnership agreement does not specify how income is to be allocated, profits and
(For question 59 – 60) loss should be allocated
DD and EE was organized and began operations of March 1, 2019. On that date, DD A. Equally.
invested P75,000 and EE invested land and building with current fair value of P40,000 and B. In proportion to the weighted average of capital invested during the period.
P50,000, respectively. EE also invested P30,000 in the partnership on November 1, 2019 C. Equitably so that partners are compensated for the time and effort expended on
because of its shortage of cash. The partnership contract includes the following behalf of the partnership.
remuneration plan: D. In accordance with their capital contributions.
DD EE
(Millan, 2016)
Annual Salary P9,000 P12,000

63. When Mill retired from the partnership of Mill, Yale, and Lear, the final settlement of 67. The partnership agreement of XX, YY and ZZ provides for the year-end allocation of net
Mill’s interest exceeded Mill’s capital balance. Under the bonus method, the excess income in the following order:
A. Was recorded as goodwill. • First, XX is to receive 10% of net income up to P200,000 and 20% over
B. Was recorded as an expense. P200,000.
C. Reduced the capital balances of Yale and Lear. • Second, YY and ZZ each are to receive 5% of the remaining income over
D. Had no effect on the capital balances of Yale and Lear. P300,000.
(Millan, 2016) • The balance of income is to be allocated equally among the three partners.

64. State the correct order of the claims on the personal assets of a partner, The partnership’s 2011 net income was P500,000 before any allocations to partners. What
I. The partner’s separate creditors amount should be allocated to XX?
II. To the other partner’s by way of contribution A. P202,000
III. The partnership creditors B. P216,000
A. I, III, II C. P206,000
B. I, II, III D. P220,000
C. III, II, I
D. II, I, III (Dayag, 2013)

(Millan, 2016) 68. RR and XX formed a partnership and agreed to divide initial capital equally, even though
RR contributed P25,000 and XX contributed P21,000 in identifiable assets. Under the
65. It is the change in the relation of the partners caused by any partner ceasing to be associated bonus approach to adjust the capital accounts. XX’s unidentifiable assets should be debited
in the carrying on of the business. for:
A. Dissolution A. P11,500
B. Liquidation B. P 4,000
C. Incorporation C. P 2,000
D. Break-up D. P 0
(Millan, 2016) (Dayag, 2013)
66. MM, NN, and OO are partners with capital balances on December 31, 2012 of P300,000, 69. As of December 31, 2012, the books of Ton Partnership showed capital balances of: T,
P300,000 and P200,000, respectively. Profits are shared equally. OO wishes to withdraw P40,000; O, P25,000; N, P5,000. The partner’s profit and loss ratio was 3:2:1, respectively.
and it is agreed that OO is to take certain equipment with second-hand value of P50,000 The partners decided to liquidate and they sold all non-cash assets for P37,000. After
and a note for the balance of OO’s interest. The equipment are carried on the books at settlement of all liabilities amounting P12,000, they still have cash of P28,000 left for
P65,000. Brand new equipment may cost P80,000. Compute for: (1) OO’s acquisition of distribution. Assuming that any capital debit balance is uncollectible, the share of T in the
the second-hand equipment will result to reduction in capital; (2) the value of the note that distribution of the P28,000 cash would be:
will OO get from the partnership’s liquidation. A. P17,800
A. (1) P 15,000 each for MM and NN, (2) P150,000 B. P18,000
B. (1) P5,000 each for MM, NN and OO, (2) P145,000 C. P19,000
C. (1) P5,000 each for MM, NN and OO, (2) P195,000 D. P17,000
D. (1) P7,500 each for MM and NN, (2) P145,000
(Dayag, 2013)
(Dayag, 2013)
70. CC, PP and AA, accountants, agree to form a partnership and to share profits in the ratio
of 5:3:2. They also agreed that AA is to be allowed a salary of P28,000, and that PP is to
be guaranteed P21,000 as his share of the profits. During the first year of operation, income A. I, III, II
from fees are P180,000, while expenses total P96,000. What amount of net income should B. I, II, III
be credited to each partner’s capital account? C. III, II, I
A. CC, P28,000, PP, P16,800, AA, P11,200 D. II, I, III
B. CC, P25,000, PP, P21,000, AA, P38,000
C. CC, P24,000, PP, P22,000, AA, P38,000
(Milan, 2016)
D. CC, P25,000, PP, P21,000, AA, P39,000
(Dayag, 2013)
74. According to the Philippine Civil Code, if only the shares of each partner in the profits
71. Allen retired from the partnership of Allen, Beck and Chale. Allen’s cash settlement from has been agreed upon, the share of each in the losses shall be
the partnership was based on new goodwill determined at the date of retirement plus the
A. equally
carrying amount of the other net assets. As a consequence of the settlement, the capital
accounts of Beck and Chale were decreased. In accounting for Allen’s withdrawal, the B. equally, but the industrial partner shall not share in the loss
partnership could have used the:
C. the same as the sharing in profits
BONUS METHOD GOODWILL METHOD D. the same as the sharing in profits. However, the industrial partner shall not
A. No Yes share in the loss.
B. No No
(Milan, 2016)
C. Yes Yes
D. Yes No
(Milan, 2016)

72. Which of the following has the least priority of payment in case of partnership 75. Which of the following is not considered a legitimate expense of a partnership?
liquidation?
A. Supplies used in the partners’ offices.
A. Priority claims such as artisans. Government, liquidation expenses B. Depreciation on assets contributed to the partnership by partners.
B. Secured creditors to the extent of covered by the proceeds from the sale of pledged
C. Salaries for management hired to run the business.
assets.
C. Unsecured credit to the extent covered by proceeds from sale of unpledged (or free) D. Interest paid to partners based on the amount of invested capital.
assets.
(Milan, 2016)
D. The partners’ capital balances.
(Milan 2016)
76. CC, PP and AA, accountants, agree to form a partnership and to share profits in the ratio
of 5:3:2. They also agreed that AA is to be allowed a salary P28,000 and that PP is to be
73. State the proper order of liquidation
guaranteed P21,000 as his share of the profits. During the first year of operation, income
I Outside creditors from fees are P180, 000, while expenses total P96,000. What amount of net income should
be credited to each partner’s capital account?
II Owners’ interests
A. CC, P28,000 PP, P16,800 AA, P11,200
III Inside creditors
B. CC, P25,000 PP, P21,000 AA, P38,000

C. CC, P24,000 PP, P22,000 AA, P38,000 (Dayag, 2015)


D. CC, P25,000 PP, 21, 000 AA, P39, 000
(Dayag, 2015)
80. RR and XX formed a partnership and agreed to divide initial capital equally, even though
77. The same information in Number 32, except the partnership had a loss of P125, 624 after RR contributed P25,000 and XX contributed P21,000 in identifiable assets. Under the
the interest and salaries to partners, by what amount should BB’s capital account change bonus approach to adjust the capital accounts. XX’s unidentifiable assets should be debited
increase(decrease)? for:
A. P (115,443) C. P (41,875)
A. 11,500 C. 2,000
B. 23,865 D. (18,010)
B. 4,000 D. 0
(Dayag, 2015)
(Dayag, 2015)

78. PP, QQ and RR, partners to a firm, have capital balances of P11, 200, P13, 000 and P5,
800, respectively, and share profits in the ratio of 4:2:1. Prepare a schedule showing how
available cash will be given to the partners as it becomes available. Who among the
partners shall be paid first with an available cash of P1, 400?
A. QQ B. No One C. RR D. PP 81. Partnership capital and drawing accounts are similar to the corporate

A. Paid-in capital, retained earnings, and dividend accounts.


B. Retained earnings account.
C. Paid-in capital and retained earnings accounts.
D. Preferred and common stock accounts.
79. The August, Albert and Gerry partnership became insolvent on January 1, 2015, and the (Gleim)
partnership is being liquidate as as practicable. In this respect the following information
for the partners has been marshaled. 82. The partnership agreement is an express contract among the partners (the owners of the
business). Such an agreement generally does not include
Capital Balances Personal Assets Personal Liabilities
August P 70,000 P 80,000 P 40,000 A. A limitation on a partner’s liability to creditors.
Albert - 60,000 30,000 50,000 B. The rights and duties of the partners.
Gerry - 30,000 70,000 30,000 C. The allocation of income between the partners.
D. The rights and duties of the partners in the event of partnership dissolution.
(Gleim)
Assume that residual profits and losses are shared equally among the three partners.
Based on this information, calculate the maximum amount that August can expect to 83. A partnership records a partner’s investment of assets in the business at
receive from the partnership liquidation is:
A. The market value of the assets invested.
A. P20, 000 C. P70,000 B. A special value set by the partners.
B. 40,000 D. 110,000 C. The partner’s book value of the assets invested.
D. Any of the above, depending upon the partnership agreement.
(RPCPA 0598) 86. If the average capital for Albion and Blaze from the above information is $112,000 and
$119,000, respectively, what will be the total amount of profit allocated after the salary
84. Assume that C has a P50,000 equity in the partnership of “A, B, and C.” Partner C arranges and interest distributions are completed?
to sell his entire interest to D for P80,000 Cash. Partners A and B agree to the admission
of D. At what amount will the equity of the incoming partner, D, be shown in the balance A. $70,000.
sheet? B. $73,100.
C. $75,000.
A. at P50,000. D. $80,000.
B. at P50,000 and the P30,000 will be divided equally among the original partners. (Beams, 2009)
C. at P80,000
D. at P80,000 and the P30,000 will represent Goodwill which will be apportioned between 87. If the average capital balances for Albion and Blaze are $100,000 and $120,000,
E. the existing equities of A and B. what will the final profit allocations for Albion and Blaze in 2006?
(RPCPA 107)
A. $50,000 and $70,000.
Albion and Blaze share profits and losses equally. Albion and Blaze receive salary allowances of B. $54,000 and $66,000.
$20,000 and $30,000, respectively, and both partners receive 10% interest on their average capital C. $70,000 and $50,000.
balances. Average capital balances are calculated at the beginning of each month balance D. $75,000 and $45,000.
regardless of when additional capital contributions or permanent withdrawals are made (Beams, 2009)
subsequently within the month. Partners’ drawings are not used in determining the average capital For number 88 to 89 refer to the problem below:
balances. Total net income for 2006 is $120,000. Bloom and Carnes share profits and losses in a ratio of 2:3, respectively. Bloom and Carnes receive
salary allowances of $10,000 and $20,000, also respectively, and both partners receive 10%
Albion Blaze interest based upon the balance in their capital accounts on January 1. Partners’ drawings are not
January 1 capital balances $ 100,000 $ 120,000 used in determining the average capital balances. Total net income for 2006 is $60,000. If net
Yearly drawings ($1,500 a month) 18,000 18,000 income after deducting the interest and salary allocations is greater than $20,000, Carnes receives
Permanent withdrawals of capital: a bonus of 5% of the original amount of net income.
June 3 ( 12,000 )
May 2 ( 15,000 ) Bloom Carnes
Additional investments of capital: January 1 capital balances $ 200,000 $ 300,000
July 3 40,000 Yearly drawings ($1,500 a month) 18,000 18,000
October 2 50,000
88. What are the total amounts for the allocation of interest, salary, and bonus, and, how
85. What is the weighted-average capital for Albion and Blaze in 2006? much over-allocation is present?

A. $100,000 and $120,000. A. $60,000 and $0.


B. $105,333 and $126,667. B. $80,000 and $20,000.
C. $110,667 and $119,583. C. $83,000 and $0.
D. $126,667 and $105,333. D. $83,000 and $23,000.
(Beams, 2009) (Beams, 2009)

89. The XYZ partnership provides a 10% bonus to Partner Y that is based upon E. Insolvency of the partnership
partnership income, after deduction of the bonus. If the partnership's income is F. Admission of a new partner in an existing partnership
$121,000, how much is Partner Y's bonus allocation? G. Assignment of an existing partner’s interest to a third person
H. Retirement of a partner
A. $11,000.
B. $11,450. CPAR Testbank
C. $11,650.
D. $12,100.
93. He refers to a partner who contributed not only money and property but also industry
(Beams, 2009)
to the newly formed partnership.
A. Industrial partner
Lara, Ives, and Jack are in the process of liquidating their partnership. Since it may take several
B. Nominal partner
months to convert the other assets into cash, the partners agree to distribute all available cash
C. Capitalist-industrial partner
immediately, except for $10,000 that is set aside for contingent expenses. The balance sheet and
D. Capitalist partner
residual profit and loss sharing percentages are as follows:
CPAR 2017 Pre-Board
Cash $ 400,000 Accounts payable $ 200,000
Other assets 200,000 Hara, capital (40%) 135,000
Ives, capital (30%) 216,000
94. It refers to a type of partnership wherein all partners are liable to the creditors pro-rata
Jack, capital (30%) 49,000
up to the extent of personal or separate assets after the partnership’s assets are
exhausted.
Total assets $ 600,000 Total liab./equity $ 600,000
A. General partnership
B. Partnership by estoppel
C. Limited partnership
90. How much cash should Ives receive in the first distribution?
D. Particular partnership
CPAR 2017 Pre-Board
A. $146,000.
B. $147,000.
C. $153,000.
95. Which of the following statements concerning the formation of partnership business is
D. $156,000
correct?
(Beams, 2009)
A. Philippine Financial Reporting Standards (PFRS) allows recognition of goodwill
arising from the formation of partnership.
91. Which of the following transactions shall not affect the capital balance of a partner? B. The juridical personality of the partnership arises from the issuance of certification of
A. Share of a partner in the partnership’s net loss registration.
B. Receipt of bonus by a partner from another partner based on the agreement C. The parties may become partners only upon contribution of money or property but not
C. Advances made by the partnership to a partner of industry or service.
D. Additional investment by a partner to the partnership D. The capital to be credited to each partner upon formation may not be the amount
actually contributed by each partner.
CPAR Testbank CPAR 2017 Pre-Board

92. Which of the following will not result to the dissolution of a partnership?
96. The partners, C and D, share profits 3:2. However, C is to receive a yearly bonus of Lucy and Annie were partner sharing profits and losses equally. Ochie was admitted as a partner
20% of the profits, in addition to his profit share. The partnership made a net income by contributing cash of P60,000 for one-third interest in the firm. They agreed to set the total
for the year of P960,000 before the bonus. Assuming C’s bonus is computed on profit capital at P210,000 after Ochie’s admission. Prior to Ochie’s admission, the old partner’s capital
after deducting said bonus, how much profit share will D receive? accounts were Lucy, P48,000, and Annie, P96,000.
A. P307,200
99. The capital balance of Annie after Ochie’s admission was
B. P320,000
A. P92,667
C. P640,000
B. P94,000
D. P160,000
C. P91,000
CPAR 2017 Pre-Board
D. P96,000
100. Assuming that Ochie will share one-fourth interest on the partnership assets the
capital balance of Annie after Ochie’s admission is
A. P96,000
97. A, B, and C are partners and share profits and losses as follows: Salaries of P40,000 to
B. 99,750
A; P30,000 to B; and none to C. If net income exceeds salaries, then a bonus is allocated
C. P99,000
to A. The bonus is 5 percent of net income after deducting salaries and the bonus.
D. P102,750
Residual profits or residual losses are allocated 10 percent to A, 20 percent to B, and
CPAR 2017 Pre-Board
70 percent to C. If net income before salaries and bonus is P140,000, how much is the
share of A?
A. P50,150
B. P43,333 101. A partnership agreement calls for allocation of profits and losses by salary
C. P46,667 allocations, a bonus allocation, interest on capital, with any remainder to be allocated by
D. P50,000 preset ratios. If a partnership has a loss to allocate, generally which of the following
CPAR 2017 Pre-Board procedures would be applied?

A. Any loss would be allocated equally to all partners.


B. Any salary allocation criteria would not be used.
C. The bonus criteria would not be used.
98. Carson and Lamb establish a partnership to operate and used-furniture business under D. The loss would be allocated using the profit and loss ratios, only.
the name of C & L Furniture. Carson contributes furniture that cost P60,000 and has a (Fisher, 2008)
fair value of P90,000. Lamb contributes P30,000 cash and delivery equipment that cost
P40,000 and has a fair value of P30,000. The partners agree to share profits and losses 102. Della Reise was admitted to a partnership. She contributed $25,000 cash plus
60% to Carson and 40% to Lamb. Calculate the peso amount of inequality that will equipment she purchased for $50,000 and which had accumulated depreciation for tax
result if the initial noncash contributions of the partners are recorded at cost rather than purposes of $20,000. The fair value of the equipment was $35,000. She also assumed 1/3
fair market value. of partnership debt of $15,000. Her beginning capital balance was $48,000. For tax
A. P30,000 purposesher partnership interest should be initially valued at
B. P10,000
C. P20,000 A. $60,000
D. P18,000 B. $48,000
CPAR 2017 Pre-Board C. $55,000
Items 99 and 100 are based on the following: D. $65,000
(Fisher, 2008)

B. $50,000
103. Under the bonus method, when a new partner is admitted to the partnership, the C. $166,667
total capital of the new partnership is equal to: D. $300,000
(Fisher, 2008)
A. the book value of the previous partnership + the fair market value of the
consideration paid to the existing partnership by the incoming partner 106. Partners Thomas, Adams and Jones have capital balances of $24,000, $45,000, and
B. the book value of the previous partnership + any necessary asset write ups from $90,000 respectively. They split profits in the ratio of 3:3:4, respectively. Under a
book value to market value + the fair market value of the consideration paid to the predistribution plan, one of the partners will get the following total amount in liquidation
existing partnership by the incoming partner before any other partners get anything:
C. the book value of the previous partnership - any asset write downs from book to
market value + the fair market value of the consideration paid to the existing A. $22,500
partnership by the incoming partner B. $30,000
D. the fair market value of the new partnership as implied by the value of the C. $40,000
incoming partner's consideration in exchange for an ownership percentage in the D. $75,000
new partnership (Fisher, 2008)
(Fisher, 2008)
107. Which of the following statements are true when comparing corporations and
104. Assume that a partnership had assets with a book value of $240,000 and a market partnerships?
value of $195,000, outside liabilities of $70,000, loans payable to partner Able of $20,000,
and capital balances for partners Able, Baker, and Chapman of $70,000, $30,000, and A. Partnership entities provide for taxes at the same rates used by corporations.
$50,000. How would the first $100,000 of available assets be distributed assuming profits B. In theory, partnerships are more able to attract capital.
and losses are allocated equally? C. Like corporations, partnerships have an infinite life.
D. Unlike shareholders, general partners may have liability beyond their capital
A. $70,000 to outside liabilities, $20,000 to Able, and the balance equally among the balances.
partners
B. $70,000 to outside liabilities and $30,000 to Able 108. Which of the following characteristics of a partnership most likely explains why a
C. $70,000 to outside liabilities, $25,000 to Able, and $5,000 to Chapman public accounting firm is organized as a partnership from a public policy viewpoint?
D. $40,000 to Able, $20,000 to Chapman, and the balance equally among the partners
(Fisher, 2008) A. A partnership is not a taxable entity.
B. A partnership is characterized by unlimited liability.
105. Partners Dalton, Edwards, and Finley have capital balances of $40,000, 90,000 and C. A partnership is characterized by a fiduciary relationship among the partners.
$30,000, respectively, immediately prior to liquidation. Total remaining assets have a book D. Salaries to the partners are not considered a component of net income.
value of $160,000, the liabilities having been paid. Among these remaining assets is a (Fisher, 2008)
machine with a fair value of $35,000. The partners split profits and losses equally. Edwards
covets the machine and is willing to accept it for $35,000 in lieu of cash. The other partners
109. The partnership agreement is an express contract among the partners (the owners
have no designs on specific assets, only cash in liquidation. How much cash, in addition to
of the business). Such an agreement generally does not include:
the machine, would be first distributed to Edwards, before any of the other partners
received anything?
A. A limitation on a partner’s liability to creditors.
B. The rights and duties of the partners.
A. $15,000
C. The allocation of income between the partners.
D. The rights and duties of the partners in the event of partnership dissolution. D. at P80,000 and the P30,000 will represent Goodwill which will be apportioned
between
(Punzalan, 2016) E. the existing equities of A and B.
(Punzalan, 2016)

110. A partnership records a partner’s investment of assets in the business at


114. Partner Morgan is personally insolvent, owing P600,000. Personal assets will only
A. The market value of the assets invested. bring P200,000 when liquidated. At the same time, Morgan has a credit balance in the
B. A special value set by the partners. partnership of P120,000. The capital amounts of the other partners total a credit balance
C. The partner’s book value of the assets invested. of P250,000. Under the doctrine of marshaling of assets, how much the personal creditors
D. Any of the above, depending upon the partnership agreement. of Morgan can collect?
(Punzalan, 2016)
A. 120,000
B. 200,000
111. When property other than cash is invested in a partnership, at what amount should C. 320,000
the noncash property be credited to the contributing partner’s capital account? D. 570,000
(Punzalan, 2016)
A. Fair value at the date of recognition.
B. Contributing partner’s original cost.
C. Assessed valuation for property tax purposes. For Numbers 115 to 116 refer to the problem below
D. Contributing partner’s tax basis. As of December 31, the books of AME Partnership showed capital balances of A-P40,000; M-
P25,000; And E-P5,000. The partners’ profit and loss ratio were 3:2:1, respectively. The partners
(Punzalan, 2016) decided to dissolve and liquidate. They sold all the non-cash assets for P37,000 cash. After
settlement of all liabilities amounting to P12,000, they still have P28,000 cash left for distribution.

112. X, Y, Z are capitalist partners and D an industrialist partner. The partnership 115. The loss on the realization of the non-cash assets was
reported a net loss of P200,000. How much is the share of D?
A. P40,000
A. 0 B. P42,000
B. 10,000 C. P44,000
C. 25,000 D. P45,000
D. 100,000
(Punzalan, 2016) (Punzalan, 2016)

116. Assuming that any partner’s capital debit balance is uncollectible, the share of A in
113. Assume that C has a P715,000 equity in the partnership of “A, B, and C.” Partner
the P28,000 cash for distribution would be
C arranges to sell his entire interest to D for P80,000 Cash. Partners A and B agree to the
admission of D.At what amount will the equity of the incoming partner, D, be shown in
the balance sheet? A. P19,000
B. P18,000
A. at P715,000. C. P17,800
B. at P50,000 and the P30,000 will be divided equally among the original partners. D. P40,000
C. at P80,000

(Punzalan, 2016) (Dayag, 2015)


120. For a partner to withdraw or retire from the partnership, the total interest of a partner
should be properly determined which includes:
117. The following balance sheet is presented for the partnership A, B and C, who share
profits and losses in the respectively ratio of 5:3:2
A.Share in the profit or loss of the partnership
B.Adjustments in assets and liabilities to reflect fair market values
Assets Liabilities and Capital
C.Loans to and from partnership
Cash Php 120,000 Liabilities Php 280,000
D.All of the above
Other Assets 1,080,000 A, Capital 560,000
(Dayag, 2015)
B, Capital 320,000
121. On December 1, 2015, EE and FF formed a partnership, agreeing to share for profits and
C, Capital 40,000
losses in the ratio of 2:3, respectively. EE invested a parcel of land that cost him P25,000.
Total Php 1,200,000 Total Php 1,200,000
FF invested P30,000 cash. The land was sold for P50,000 on the same date, thress hours
after formation of the partnership. How much should be the capital balance of EE right
Assume the three partners decided to liquidate the partnership. If the other assets are sold
after formation?
for P800,000, how should the available cash be distributed to each partner?
A B C A. P25,000
A. 280,000 320,000 40,000 B. P30,000
B. 324,000 236,000 16,000 C. P60,000
C. 410,000 230,000 0 D. P50,000
D. 412,000 228,000 0 (Dayag, 2015)

(Punzalan, 2016) 122. MM, NN, and OO are partners with capital balances on December 31, 2015 of P300,000,
P300,000 and P200,000, respectively. Profits are shared equally. OO wishes to withdraw
and it is agreed that OO is to take certain equipment with second-hand value of P50,000
118. Partners Almond, Barney and Colors have capital balances of P20,000, P50,000, and a note for the balance of OO’s interest. The equipment are carried on the books at
and P90,000, respectively. They split profits in the ratio of 2:4:4, respectively. Under a P65,000. Brand new equipment may cost P80,000. Compute for: (1) OO’s acquisition of
safe cash distribution plan, one of the partners will get the following total amount in the second-hand equipment will result to reduction in capital; (2) the value of the note that
liquidation before any other partners get anything will OO get from the partnership’s liquidation.
A. 0
B. 15,000 A. (1) P15,000 each for MM and NN, (2) P150,000
C. 40,000 B. (1) P5,000 each for MM, NN and OO, (2) P145,000
D. 180,000 C. (1) P5,000 each for MM, NN and OO, (2) P195,000
D. (1) P7,500 each for MM and NN, (2) P145,000
(Punzalan, 2016) (Dayag, 2015)

119. Methods exist for the division of partnership profits and losses 123. JJ and KK are partners who share profits and losses in the ratio of 60%: 40%, respectively.
JJ’s salary is P60,000 and P30,000 for KK. The partners are also paid interest on their
A. Equally average capital balances. In 2015, JJ received P30,000 of interest and KK, P12,000. The
B. Arbitrary ratio profit and loss allocation is determined after deductions for the salary and interest
C. Capital contribution ratio
D. All of the above
payments. If KK’s share in the residual income (income after deducting salaries and (Dayag, 2015)
interest) was P60,000 in 2015, what was the total partnership income?
A. P192,000 126. Larry, Marsha, and Natalie are partners in accompany that is being liquidated. They share
B. P345,000 profits and losses 55 percent, 20 percent, and 25 percent, respectively. When the liquidation
C. P282,000 begins, they have capital account balances of P108,000, P62,000 and P56,000,
D. P387,000 respectively. The partnership just sold equipment with a historical cost and accumulated
(Dayag, 2015) depreciation of P25,000 and P18,000, respectively for P10,000. What is the balance in
Marsha’s capital account after the transaction is completed?
124. CC, PP, and AA, accountants, agree to form a partnership and to share profits in the ratio
of 5:3:2. They also agreed that AA is to be allowed a salary of P28,000, and that PP is to A. P62,000
be guaranteed P21,000 as his share of the profits. During the first year of operation, income B. P61,400
from fees are P180,000, while expenses total P96,000. What amount of net income should C. P62,600
be credited to each partner’s capital account? D. P65,000
(Dayag, 2015)
A. CC, P28,000, PP, P16,800, AA, P11,200
B. CC, P25,000, PP, P21,000, AA, P38,000 127.Partner A first contributed P50,000 of capital into an existing partnership on March 1,
C. CC, P24,000, PP, P22,000, AA, P38,000 2015. On June 1, 2015, the partner contributed another P20,000. Withdrawals in excess of
D. CC, P25,000, PP, P21,000, AA, P39,000 P10,000 are charged to the partner’s capital account. The annual weighted-average capital
(Dayag, 2015) balance is

125.The following condensed balance sheet is presented for the partnership of AA, BB, and A. P62,000
CC, who share profits and losses in the ratio of 4:3:3, respectively: B. P51,667
C. P60,000
Cash P160,000 D. P48,333
Other Assets 320,000
Total P480,000 (Dayag, 2015)
128. (1) All assets contributed to the partnership are recorded by the partner at their agreed
Liabilities P180,000 values.
AA, capital 48,000 (2) All liabilities that the partnership assumes are recorded at their net present values.
BB, capital 216,000
CC, capital 36,000 A. Only the first statement is correct
Total P480,000 B. Only the second statement is correct
C. Both statements are correct
The partners agreed to dissolve the partnership after selling the other assets for P200,000. D. Both statements are incorrect
Upon dissolution of the partnership, AA should have received (Dayag, 2015)

A. P0
129. If a partnership has only non-cash assets, all liabilities have been properly
B. P48,000
disbursed, and no additional liquidation expenses are expected, the maximum potential
C. P72,000
loss to the partnership in the liquidation process is:
D. P84,000

A. The fair market value of the non-cash assets 133. Which of the following statements is correct regarding a partner’s debit capital
B. The book value of the non-cash assets balances?
C. The estimated proceeds from the sale of the assets less the book value of the
non-cash assets. A. The partner should make contributions to reduce the debit balance to whatever extent
D. None of the above. possible.
(RESA Pre-Board July 2017) B. If contributions are not possible, the other partners with credit capital balances will be
allocated a portions of the debit balance based on their proportionate profit-and-loss-
130. In partnership, sharing percentages.
C. Partners who absorb another’s debit capital balance have a legal claim against the
A. Management consists of the board of directors deficient partner.
B. Profits are always divided equally among partners D. All of these statements are correct.
C. Dissolution results when a partner leaves the partnership (RESA Pre-Board July 2017)
D. No partner is liable for more than a proportion of the company’s debt
(RESA Pre-Board April 2016)
134. Following is the balance sheet of the ABCD Partnership at March 31, 2018, when
131. Mr. MAC is admitted into the partnership of Do and Nald by investing cash the partnership is to be liquidated:
equivalent to ¼ of their capital. Which of the following is true after the admission of
Cash P 6, 000 Liabilities P 12, 400
Mr. MAC?
Other Assets 126, 000 A, Loan 12, 000
B, Loan 14, 400
A. Assets of the partnership will increase
D, Loan 9, 600
B. Total partner’s equity remain the same
A, Capital – 25% 16, 200
C. Do and Nald capital decreased by ¼.
B, Capital – 25% 12, 000
D. Assets of the partnership will remain the same
C, Capital – 25% 37, 700
(RESA Pre-Board July 2017)
D, Capital – 25% 17, 700
132. Under the bonus method, when a new partner is admitted to the partnership, the
total capital of the new partnership is equal to:
During the month of April 2018, assets having a book value of P 18, 000 are sold at a loss of
P 2, 400. Liquidation expenses of P 600 are paid as well as P 7, 200 of the liabilities. Of the
A. The book value of the previous partnership plus the fair market value of the liabilities shown in the balance sheet, P 240 represents salary payable to D and P 160 represents
consideration paid to the existing partnership by the incoming partner. salary payable to C.
B. The book value of the previous partnership plus any necessary asset write-ups from
book value to market value plus the fair market value of the consideration paid to the On April 30, 2018 cash to be distributed to A, B, C and D as follows:
existing partnership by the incoming partner.
C. The book value of the previous partnership minus any asset write downs from book to A B C D
market value plus the fair market value of the consideration paid to the existing A. P 0 P 0 P 0 P 9, 000
partnership by the incoming partner. B. P 1, 950 P 1, 950 P 1, 950 P 1, 950
D. The fair market value of the new partnership as implied by the value of the incoming C. P 0 P 0 P 0 P 1, 950
partner’s consideration in exchange for an ownership percentage in the new D. P 0 P 0 P 9, 000 P 0
partnership. (RESA Pre-Board July 2017)
(RESA Pre-Board July 2017)
135. Cheryl is the manager of a local store. She is also a partner in the company and she 138. XX, YY, and ZZ, a partnership formed on January 1, 2018 had the following initial
receives a bonus as part of the profit and loss allocation. Cheryl’s bonus is based on the investment:
increase in revenues recorded during the period. The bonus arrangement is that Cheryl XX ………………………………………P 170, 000
receives 1 percent of net income for every full percentage point growth for revenues in YY ………………………………………. 255, 000
excess of a 5 percent revenue growth. During the most recent period, revenues grew ZZ ………………………………………. 382, 500
from P500, 000 to P540, 000 and net income grew from P 98, 000 to P 120, 000. How
much bonus does Cheryl receive for this period? The partnership agreement states that the profits and losses are to be shared equally by the
partners after consideration is made for the following:
A. P 1, 100 - Salaries allowed to partners: P102, 000 for XX, P81, 600 for YY, and P61, 200 for ZZ.
B. P 3, 600 - Average partners’ capital balances during the year shall be allowed 10%.
C. P 2, 000
D. P 6, 000 Additional information:
- On June 30, 2018, XX invested an additional P102, 000.
(RESA Pre-Board July 2017)
- ZZ withdrew P119, 000 from the partnership on September 30, 2018.
Use the following information for questions 8 and 9: - Share the remaining partnership profit was P 8, 500 for each partner.

Cleary, Wasser, and Nolan formed a partnership on January 1, 20x4, with investments of The total partnership capital on December 31, 2018 was:
P 100, 000, P 150, 000, and P 200, 000, respectively. For division of income, they agreed A. P 688, 500
to (1) interest of 10% of the beginning capital balance each year, (2) annual compensation B. P 1, 141, 550
of P 10, 000 to Wasser and (3) sharing the remainder of the income or loss in a ratio of C. P 816, 000
20% for Cleary and 40% each for Wasser and Nolan. Net income was P 150, 000 in 20x4 D. d. P 1, 143, 675
and P 180, 000 in 20x5. Each partner withdrew P 1, 000 for personal use every month (RESA Pre-Board July 2017)
during 20x4 and 20x5.
139. At the time of partnership liquidation, which credits shall be settled first?
136. What was Wasser’s share of income for 20x4?
A. Those amount owing to third persons.
A. P 63, 000 B. Those amount owing to partners other than capital contribution and share in profit.
B. P 53, 000 C. Those amount owing to partners with respect to capital contribution.
C. P 58, 000 D. Those amount owing to partners with respect to share in profit.
D. P 29, 000 (CPAR Reviewer, 2017)
E. P 51, 000 140. How should the net profit or net loss of the partnership be divided among the
(RESA Pre-Board April 2016) partners, whether capitalist or industrial?

137. What was Wasser’s capital balance at the end of 20x5? A. In accordance with their capital contribution ratio.
A. P 201, 000 B. In accordance with just and equitable sharing taking into account the circumstances of
B. P 263, 520 the partnership.
C. P 264, 540 C. Equally
D. P 304, 040 D. In accordance with the partnership agreement.
E. P 313, 780 (CPAR Reviewer, 2017)
(RESA Pre-Board April 2016)

141. At the date of partnership formation of a partnership, the amount credited to A’s On January 2, 2017, the partnership was able to sell the investment property for
capital is less than the fair value of the property contributed. Which is the most valid P2,000,000. How much cash shall be contributed by JM if the articles of co-partnership
reason? provide that Toni will have 60% interest in the partnership?

A. The property contributed by A is impaired. A. 500,000


B. The property contributed by A has been subjected to positive asset revaluation. B. 700,000
C. Bonus has been given by partner A to the other partners. C. 800,000
D. Goodwill arising from partnership formation has been recognized. D. 600,000
(CPAR Reviewer, 2017)
(CPAR Reviewer, 2017)
142. When a new partner is admitted to an existing partnership through the purchase of
a portion of existing interest of an incumbent partner, which statement is correct? 145. On January 1, 2017, Yazzi, Angel and Nadine organized YAN partnership by
investing P5M, 2M and P3M for capital interest ratio of 4:5:1 respectively. Nadine has
A. The total capital of the old and new partnership will be the same. been appointed as managing partner. During year 2017, YAN partnership reported net
B. The partnership will recognize gain or loss on the difference between the amount paid income of P3,000,000. Their profit/loss distribution and drawing agreement are presented
and capital transferred. below:
C. Goodwill may be recognized by virtue of the admission.
D. There will be increase in the total assets of the partnership equivalent to the amount • 20% interest on beginning capital
paid by the newly admitted partner. • P10,000, P20,000 and P50,000 monthly salary, respectively
(CPAR Reviewer, 2017) • 25% bonus of net income after interest and salary to managing partner
• The remainder will be divided equally among the partners.
143. At the time of retirement, a retiring partner receives more than the amount of his • The partners must withdraw at the end of the year 50% of their share in net income for
capital contribution while the remaining partners capital increase after the retirement. the period.
Which of the following is most valid reason?
What is the capital balance of Nadine on December 31, 2017?
A. Goodwill during retirement is recognized.
B. Asset revaluation is recognized. A. 1,410,000
C. Bonus is given by retiring partner to remaining partners. B. 3,410,000
D. Bonus is given by the remaining partners to retiring partner. C. 1,610,000
(CPAR Reviewer, 2017) D. 3,610,000
(CPAR Reviewer, 2017)
144. On January 1, 2017, Toni, Abbie and JM entered into articles of co-partnership for 146. On December 31, 2017, the capital balance of partners Cristy, Paula and Ara of
the operation of TAJ computer shop. Toni contributed investment property with assessed CPA Partnership are P1M, P3M and P6M, respectively with profit or loss agreement ratio
value of P1,700,000 subject to mortgage payable of P500,000 to be assumed by the of 4:1:5. On January 1, 2018, Cristy decided to retire and received P400,000 from the
partnership. Abbie contributed computer equipment with cost of P600,000 with partnership.
accumulated depreciation of P200,000. The fair market value of the computer equipment
is P300,000. If the assets of the partnership are not properly valued at the time of retirement, how much
is the capital balance of Paula after the retirement of Cristy?

A. 2,900,000
B. 2,850,000 149. On December 1, 2011, EE and FF formed a partnership, agreeing to share for profits and
C. 3,100,000 losses in the ratio of 2:3, respectively. EE invested a parcel of
D. 3,150,000 land that cost him P25,000. FF invested P30,000 cash. The land was sold for
(CPAR Reviewer, 2017) P50,000 on the same date, three hours after formation of the partnership.
How much should be the capital balance of EE right after formation?
Numbers 147 and 148
On December 31, 2017, the Statement of Financial Position of DEF with profit or loss ratio A. P25,000
of 4:1:5 is presented below: B. 30,000
C. 20,000
D. 50,000
Cash 2M Liability to third person 4M
Noncash asset 8M D, capital 3.5M
E, capital 1.5M
F, capital 1M 150. MM, NN, and OO are partners with capital balances on December 31
2011 of P300,000, P300,000 and P200,000, respectively. Profits are shared
On January 31, 2018, DEF partnership has been subjected to installment liquidation. As of equally. OO wishes to withdraw and it is agreed that OO is to take certain
January 31, 2018, the following data concerning liquidation are provided: equipment with second-hand value of P50,000 and a note for the balance
of OO's interest. The equipment are carried on the books at P65,000. Brand
➢ Noncash asset with book value of P6M has been sold at a loss of P2M. new equipment may cost P80,000. Compute for: (1) OO's acquisition of
➢ Liquidation expense amounting to P400,000 has been incurred for the month of the second-hand equipment will result to reduction in capital; (2) the value
January. of the note that will OO get from the partnership's liquidation.
➢ P600,000 cash has been withheld for future liquidation expense.
➢ P3M liability has been paid. A. (1) P15.000 each for MM and NN,(2) P150,000.
B. (1) P5,000 each for MM, NN and OO,(2) P145,000
147. What is F’s share in the maximum possible loss on January 31, 2018? C. (1) P5,000 each for MM. NN and OO,(2) P195,000
D. (1) P7,500 each for MM and NN,(2) P145,000.
A. 1,300,000
B. 1,000,000
C. 1,500,000
D. 500,000

151. JJ and KK are partners who share profits and losses in the ratio of 60%: 40%
148. What is the amount received by E on January 31, 2018?
respectively. JJ's salary is P60,000 and P30,000 for KK. The partners are also
paid interest on their averdge capital balances. In 2011, JJ received P30.000
A. 300,000
of interest and KK, P12,000. The profit and loss allocation is determined
B. 700,000
after deductions for the salary and interest payments. If KK's share in the
C. 1,000,000
residual income (income after deducting salaries and interest) was P60,000
D. 0
in 2011,

What was the total partnership income?

A. P192,000
The balance of income is to be allocated equally among the three partners.
B. 345,000
The partnership's 2011 net income was P500,000 before any allocations to
C. P282,000
partners. What amount should be allocated to XX?
D. 387,000

A. P202,000
B. 216,000
C. P206,000
D. 220,000
152. Lancelot is trying to decide whether to accept a salary of P40.000 or a
salary of P25.000 plus a bonus of 10% of net income after salary and bonus
as a means of allocating profit among the partners. Salaries traceable to
155. On April 30, 2011, XX, YY and ZZ formed a partnership by combining their
the other partners are estimated to be P100,000. What amount of income
separate business proprietorships. XX contributed cash of P75,000. YY
would be necessary so that Lancelot would consider the choices to be contributed property with a P54,000 carrying amount, a P60,000 original
equal cost, and P120,000 fair value. The partnership accepted responsibility for
the P52,500 mortgage attached to the property. ZZ contributed equipment
A. P165,000
with a P45,000 carrying amount, a P112,500 original cost, and P82,500 fair
B 290,000
value. The partnership agreement specifies that profits and losses are to
C. P265,000
be shared equally but is silent regarding capital contributions. Which
D. 305,000
partner has the largest April 30, 2011, capital balance?
153. Merlin, a partner in the Camelot Partnership, has a 30% participation in
partnership profits and losses. Merlin's capital account has a net decrease A. XX
of P.200.000 during the calendar year 2011. During 2011, Merlin withdrew B. YY
C. ZZ
P2.600,000 (charged against his capital account) and contributed property
D. All capital account balances are equal
valued at P500,000 to the partnership. What was the net income of the
Camelot Partnership for year 2011? 156. The Partnership has the following accounting amounts:

A. P3,000,000
(1) Sales P70,000
B. 4,666,667
(2) Cost of Goods Sold P40,000
C. P 7,000,000
(3) Operating Expenses P10,000
D. 11,000,000 (4) Salary allocations to partners P13,000
(5) Interest paid to banks P2,000
(6) Partners' withdrawals P8,000
The partnership net income (loss) is:

154. The partnership agreement of XX, YY & ZZ provides for the A. P20,000
allocation of net income in the following order: B 18,000
C. P 5,000
First, XX is to receive 10% of net income up to P200.000 and 20% over P200,000.
Second, YY and ZZ each are to receive 5% of the remain income over P300,000
157. The capital accounts of the partnership of NN, vv, and JJ on lune C. Admission of a new partner by purchase of existing partner’s interest below its
are presented below with their respective profif and loss ratios: (P139,200 1/2 , 208,800 1/3 , book value
96,000 1/6) On June 1, 2011, LL is admitted to the partnership when LL purchased, for D. Retirement of an existing partner with payment of above the book value of such
P132,000, a proportionate interest from NN and JJ in the net assets and interest
profits of the partnership. As a result of a transaction LL acquired a (CPAR Final Pre-board Examination May 2017)
fifth interest in the net assets and profits of the firm. What is the combined gain realized by
NN and JJ upon.the sale of a portion of their interest in 160. A, B, and C are partners with average capital balances during 2017 of P472,500, P238,650
the partnership to LL? and P162,350; respectively. The partners receive 10% interest on their average capital
balances; after deducting salaries of P122,325 to A and P82,625 to C, the residual profit or loss
A. P 0 is divided equally.
B. 43,200
C P62,400 In 2017, the partnership had net loss of P125,624 before interest and salaries to partners.
D. 82,000 What amount should A and C capital change respectively?

A. P40,844 decrease and P31,237 decrease


158. On January 31, 2011, partners of Lon, Mac & Nan, LLP, had the following B. P30,267 increase and P40,448 decrease
loan and capital account balances (after closing entries for January): C. P29,476 increase and P17,536 increase
D. P28,358 increase and P32,458 increase
Loan receivable from Lon 20,000 dr
(CPAR Final Pre-board Examination May 2017)
Loan payable to Nan 60,000 cr
Lon, capital 30,000 dr
161. A partner was admitted in an existing partnership through investment of cash equivalent to
Mac, capital 120,000 dr
Nan, capital 70.000cr ¼ of the new capitalization. If the capital balance of the old partners increases, what is the most
valid reason under Philippine GAAP?
The partnership's income sharing ratio was Lon, 50%; Mac, 20%, and Nan,
30%. On January 31, 2011, Ole was admitted to the partnership for a 20% A. Asset revaluation of existing partnership’s assets
interest in total capital of the partnership in exchange for an investment B. Impairment loss of existing partnership’s assets
of P40,000 cash. Prior to Ole's admission, the existing partners agreed to C. Recognition of goodwill of existing partnership
increase the carrying amount of the partnership's inventories to current D. Receipt of bonus from the new partner
fair value, a P60,000 increase. The capital account to be credited to Ole: (CPAR Final Pre-board Examination May 2017)

A. P60,000 162. Which of the following statements concerning the formation of partnership business is
B. P40,000 correct?
C. P52,000
D. P46,000
A. PFRS allows recognition of goodwill arising from the formation of partnership
B. The juridical personality of the partnership arises from the issuance of
159. Which of the following transactions will not affect the total equity of the partnership? certification of registration
C. The parties may become partners only upon contribution of money or property but
A. Recognition of impairment loss in case of admission of a new partner not of industry or service
B. Withdrawal of a partner D. The capital to be credited to each partner upon formation may not be the amount
actually contributed by each partner

(CPAR Final Pre-board Examination May 2017) C. P25,000,000


D. P32,000,000
163. Regina, Jessica and Nataly entered into a contract of partnership with a total capital (CPAR Final Pre-board Examination May 2017)
contribution of P5,000. The parties failed to register its articles of co-partnership with the
Securities and Exchange Commission. Which of the following statements is correct? 166. On January 1, 2017, Angel, Bea and Colleen formed ABC & Co., a general professional
partnership for the exercise of their common profession. Angel contributed a building with a
A. The contract of partnership is void because the law provides that when the capital cost of P5M and accumulated depreciation of P4M. Based on the city assessor’s records, the
contribution is at least P3,000 it must be registered with Securities and Exchange building has an assessed value of P2M. The building has an annotated mortgage payable
Commission amount to P500,000 to be assumed by the partnership.
B. The contract of partnership will bind third persons On the other hand, Bea contributed 10,000 shares of stocks with par value of P200/share
C. The contract of partnership remains to be valid and prevailing quoted price of P300/share. On January 2, 2017, the building contributed by
D. The partnership does not obtain juridical personality for failure to register with Angel was sold for P5.5M. If Colleen wants to have 20% capital interest in the newly
Securities and Exchange Commission. formed partnership, how much cash shall be contributed by her?
(CPAR Final Pre-board Examination May 2017)
A. P875,000
164. In the absence of agreement as to distribution of profit, how shall the partnership profit be B. P1,125,000
distributed to the partners? C. P2,125,000
D. P2,000,000
A. The industrial partner shall receive a share equivalent to the least share of a (CPAR Final Pre-board Examination May 2017)
capitalist partner while the capitalist partners shall share based on capital
contribution ratio. 167. On January 1, 2014, AB and QR agreed to form a partnership. The following are their
B. The industrial partner shall receive a just and equitable share and the remainder assets and liabilities:
shall be distributed to the capitalist partners on the basis of capital contribution Accounts AB QR
ratio Cash 136,000 76,000
C. The profit shall be distributed on the basis of loss contribution ratio which may Accounts Receivable 88,000 48,000
have been agreed upon by the partners Inventories 304,000 364,000
D. The profit shall be distributed equally to all partners including the industrial Machinery 480,000 440,000
partner Accounts Payable 216,000 144,000
Notes Payable 140,000 60,000
(CPAR Final Pre-board Examination May 2017)
AB decided to pay off his notes payable from his personal assets. It was also agreed that
165. On July 1, 2016, Anne, Bianca and Carla formed a business partnership to be operated as QR inventories were overstated by P24,000 and AB machinery was over depreciated by
an advertising agency. Anne contributed P10M cash while Bianca shall have a capital credit P20,000. QR is to invest/withdraw cash in order to receive a capital credit that is 20% more
of P6M upon receipt of bonus of P1M from Anne based on the provision in Articles of Co- than AB’s total net investment in the partnership.
Partnership. The terms of the agreement provide that Anne and Bianca shall have a combined
40% capital interest in the newly formed partnership. What is the capital contribution made by How much cash will be presented in the partnership’s statement of financial position?
Carla to the partnership?
A. 486,400
A. P24,000,000 B. 546,300
B. P22,500,000 C. 250,400
D. 640,300 (Punzalan, 2015)
(CPAR Final Pre-board Examination May 2017)
171. Which of the following is not a characteristic of a partnership?
168. On December 1, 2014, MG and AN are combining their separate businesses to form a A. Limited liability
partnership. Cash and noncash assets are to be contributed. The noncash assets to be B. Limited life
contributed and the liabilities to be assumed are as follows: C. Mutual agency
D. Ease of formation
MG AN (Punzalan, 2015)
Book value Fair value Book value Fair value
Accounts Receivable 250,000 262,500 200,000 195,000 172. Abel and Carr formed a partnership and agreed to divide initial capital equally, even though
Inventory 400,000 450,000 200,000 207,500 Abel contributed P100,000 and Carr contributed P84,000 in identifiable assets. Under the
bonus approach to adjust the capital accounts, Carr’s unidentifiable assets should be debited
PPE 1,000,000 912,500 862,500 822,500
for
Accounts Payable 150,000 150,000 112,500 112,500
A. 46,000
B. 16,000
MG and AN are to invest equal amount of cash such that the contribution of MG would be 10%
C. 8,000
more than the investment of AN. What is the amount of cash presented on the partnership’s
D. 0
statement of Financial Position on December 1, 2014?
(Punzalan, 2015)
A. 5,025,000
173. Alder, Benson and Carl are capitalist partners and Denver, an industrial partner. The
B. 5,500,000
partnership reported a net loss of P100, 000. How much is the share of Denver in the reported
C. 5,750,000
net loss?
D. 4,950,000
A. 0
(CPAR Final Pre-board Examination May 2017)
B. 10,000
C. 25,000
169. After the admission of a new partner, the total partnership capital increased by the fair
D. 100,000
value of the new partner’s net contributions to the partnership. The admission was accounted
(Punzalan, 2015)
for
A. Under the goodwill method
174. XYZ Partnership provided for the following in their distribution of profits and losses:
B. Under the bonus method
First: X to receive 10 % of net income up to P100,000 and 20% of the amount in excess
C. As a purchase of interest
thereof.
D. As an investment in the partnership
Then: Y and Z are each to receive 5% of the remaining income in excess of P150,000
(Millan, 2016)
after X’s share.
Finally: The balance is to be distributed equally to the three partners.
170. If a new partner acquires a partnership interest directly from the partners rather than from
the partnership itself,
If the partnership earned a net income of P250,000, what is the total share of Partner X?
A. No entry is required.
B. The partnership should be revalued.
A. 100,000
C. The existing partners’ capital accounts should be reduced and the new partner’s
B. 108,000
account increased.
C. 110,000
D. The partnership has undergone a quasi-reorganization.

D. 130,000 A. 270,000
(Punzalan, 2015) B. 260,000
C. 215,000
175. After incurring losses resulting from very unprofitable operations, the Goh Kong Wei D. 0
Partnership decided to liquidate when the partners’ capital balances were: (Punzalan, 2015)

Goh, Capital (40%) P80,000 177. Partner Morgan is personally insolvent, owing P600,000. Personal assets will only bring
Kong, Capital (40%) 130,000 P200,000 when liquidated. At the same time, Morgan has a credit balance of P120,000. The
Wei, Capital (20%) 96,000 capital amounts of the other partners total a balance of P250,000. Under the doctrine of
marshalling of assets, how much the personal creditors of Morgan can collect?
The noncash assets were sold in installment. Available cash were distributed to partners in A. 120,000
every sale of noncash assets. After the second sale of noncash assets, the partners received B. 200,000
the same amount of cash in the distribution. And from the third sale of noncash assets, cash C. 320,000
available for distribution amounts to P28,000, and unsold noncash assets has a book value D. 570,000
of P12,500. Using cash priority program, what amount did Wei receive in the third (Punzalan, 2015)
installment of cash?
178. The partnership agreement of Reid and Simm provides that 10% per year is to be credited
A. 11,600 to each partner on the basis of weighted-average capital balances. A summary of Simm’s
B. 8,000 capital account for the year-ended December 31, 2014, is as follows:
C. 5,600
D. 0 Balance, January 1 P140, 000
(Punzalan, 2015) Additional Investment, July 1 40, 000
Withdrawal, August 1 (15, 000)
176. The condensed balance sheet of Adams & Gray, a partnership, at December 31, 2014, Balance, December 31 165, 000
follows:
What amount of interest should be credited on Simm’s capital account for 2014?
Current assets P250,000 A. 15,250
Equipment (net) 30,000 B. 15,375
Total assets P280,000 C. 16,500
D. 17,250
Liabilities P20,000 (Punzalan, 2015)
Adams, Capital 160,000
Gray, Capital 100,000 179. The fact that salaries paid to partners are not a component of partnership income is
Total liabilities and capital P280,000 indicative of
A. A departure from generally accepted accounting principles
B. Being characteristic of the entity theory
On December 31, 2014, the fair values of the assets and liabilities were appraised at
C. Being characteristic of the proprietary theory
P240,000 and P20,000, respectively, by an independent appraiser. On January 2, 2015, the
D. Why partnerships are characterized by unlimited liability
partnership was incorporated and 1,000 shares of P5 par value common stock were issued.
(Punzalan, 2016)
Immediately after the incorporation, what amount should the new corporation report as
additional paid in capital?
180. The doctrine of marshalling of assets On January 1, 2014, L, M, and N formed a partnership with capital contributions of P625,000;
P750,000; and P937,500, respectively. The partners agreed that profit and loss would be allocated
A. Is applicable only if the partnership is insolvent as follows: P75,000 salary to each partner, 3% interest on initial capital contributions, the
B. Allows partners to first contribute personal assets to unsatisfied partnership creditors remainder divided in the ratio 2:4:4, respectively to L, M, and N. The partnership generated income
C. Is applicable if either the partnership is insolvent or individual partners are insolvent amounting to P375,000 for the year 2014. During 2014, the following partnership errors were
D. Amount owed to personal creditors and to partnership for debit capital balances are discovered before the distribution of profit:
shared proportionately from the personal assets of the partners
• In 2014, a purchase of piece of equipment costing P50,000 was expensed. The equipment has
(Punzalan, 2016)
an estimated life of ten years with equal service potential each year.
• On December 31, 2014, ending inventory was understated by P50,000.
181. If goodwill is traceable to the incoming partner, the new partner's capital balance equals
On January 1, 2015, N decided to retire from the partnership.
A. the fair market value of consideration paid by the incoming partner
183. If the balance of the capital of L after retirement amounts to P770,000, how much is the
B. the book value of the older partnership divided by the existing partners' ownership
settlement to N for his retirement?
percentage in the new partnership minus the book value of the old partnership.
C. incoming partner's ownership percentage multiplied by the capital of the new
A. P1,120,000 C. P1,085,000
partnership
B. P1,062,500 D. P1,110,875
D. none of the above.
(RESA, 2014)
(Guerrero, 2014)
184. If the balance of the capital of M after retirement amounts to P890,000, how much is the
settlement to N for his retirement?
AY and AN are partners who have the agreement to share profit and loss in the following manner:
A. P1,127,500
AY AN B. P1,090,500
Annual salaries 261,000 259,000 C. P1,231,500
Interest on average balances 5% 10% D. P1,152,500
Bonus (based on net income after salaries and interest) 10% (RESA, 2014)
Remainder 50% 50%
On December 1, 2014, MG and AN are combining their separate businesses to form a partnership.
During the year ended December 31, 2014, the partnership generated a profit of P575,000 before Cash and noncash assets are to be contributed. The noncash assets to be contributed and the
any deductions. AY’s and AN’s average capital balances for the year are P600,000 and P300,000, liabilities to be assumed are as follows:
respectively. Income is distributed to the partners only as far as it is available. MG AN
Book value Fair value Book value Fair value
182. How much is the total share of AN in the net income for the year ended 2014? Accounts Receivable 250,000 262,500 200,000 195,000
Inventory 400,000 450,000 200,000 207,500
A. P286,500 C. P288,500 PPE 1,000,000 912,500 862,500 822,500
B. P287,500 D. P295,665 Accounts Payable 150,000 150,000 112,500 112,500
(RESA, 2014)
MG and AN are to invest equal amount of cash such that the contribution of MG would be 10%
more than the investment of AN.

The partnership of CD, AY, and GP decided to liquidate their partnership on May 31, 2013. Before
185. What is the amount of cash presented on the partnership’s statement of Financial Position liquidating and sharing of net income, their capital balances are as follows: CD (30%) P875,000,
on December 1, 2014? AY (30%) P630,000, and GP (40%) P770,000. Net income from January 1 to May 31 is P420,000.
(RESA,2014) Liabilities of the partnership amounted to P735,000 and its total assets include cash amounting to
A. P2,762,500 P245,000. Unsettled liabilities are P385,000. CD invested additional cash enough to settle their
B. P2,512,500 partnership’s indebtedness. AY is personally solvent, GP is personally insolvent, and CD becomes
C. P5,525,000 insolvent after investing the cash needed by the partnership.
D. P5,025,000
187. How much were the partnership’s non-cash sold for?
A. P157,500 C. P105,000
On December 1, 2014, MV and CD agreed to invest equal amounts and share profits equally to B. P3,080,000 D. P525,000
form a partnership. MV invested P3,120,000 cash and a piece of equipment. CD invested some
assets which are shown on the next page: 188. How much will CD receive as a result of their liquidation?
A. P385,000
Book value
B. 0
Accounts Receivable 400,000 C. P315,000
D. P462,000
Inventory 1,120,000
(RESA, 2014)
Machineries, net 2,240,000
189. On April 30, 2016, Al, Ben, and Ces formed a partnership by combining their separate
Intangibles, net 920,000 business proprietorships. Al contributed cash of P50,000. Ben contributed property with a
P36,000 carrying amount, a P40,000 original cost, and P80,000 fair value. The partnership
accepted responsibility for the P35,000 mortgage attached to the property. Ces contributed
The assets invested by CD are not properly valued, P32,000 of the accounts receivable are proven equipment with a P30,000 carrying amount, a P75,000 original cost, and P55,000 fair value.
uncollectible. Inventories are to be written down to P1,040,000. Included in the machineries is an The partnership agreement specifies that profits and losses are to be shared equally but is
obsolete apparatus acquired for P384,000 with an accumulated depreciation balance of P336,000. silent regarding capital contributions. Which partner has the largest capital account balance
Part of the intangibles is a patent with a carrying value of P56,000 which was sued upon by a at April 30, 2016?
competitor. CD unsuccessfully defended the case and the final decision of the court was released A. Al
on November 29, 2014. B. Ben
C. Ces
D. All capital balances are equal
(Punzalan, 2015)
186. What is the fair value of the equipment invested by MV?
190. A partnership records a partner’s investment of assets in the business at
A. P1,400,000 C. P1,344,000 A. The market value of the assets invested.
B. A special value set by the partners.
B. P968,000 D. P1,560,000 C. The partner’s book value of the assets invested.
(RESA, 2014) D. Any of the above, depending upon the partnership agreement.
(RPCPA 0598) B. Shows the successive losses necessary to eliminate the capital accounts of partners
191. In the Adel-Brick partnership, Adel and Brick had a capital ratio of 3:1 and a profit and (assuming no contribution of personal assets by partners).
loss ratio of 2:1, respectively. The bonus method was used to record Colter’s admittance as a C. Indicates the distribution of successive amounts of available cash to each partner.
new partner. What ratio would be used to allocate, to Adel and Brick, the excess of Colter’s D. Assumes contribution of personal assets by partners unless there is a substantial
contribution over the amount credited to Colter’s capital account? presumption of personal insolvency by the partners.
(Gleim)
A. Adel and Brick’s new relative capital ratio. 195. The following condensed balance sheet is presented for the partnership of Axel, Barr, and
B. Adel and Brick’s new relative profit and loss ratio. Cain, who share profits and losses in the ratio of 4:3:3, respectively:
C. Adel and Brick’s old capital ratio. Cash P100,000
D. Adel and Brick’s old profit and loss ratio. Other assets 300,000
(AICPA 0r92 T-35) Total 400,000
192. The final cash distribution to the partners in a partnership in liquidation should be made in
accordance with Liabilities P150,000
Axel, Capital 40,000
A.Balances of the partners’ capital accounts. Barr, Capital 180,000
B.Partners’ profit and loss sharing ratio. Cain, Capital 30,000
C.Ratio of capital contributions made by the partners. Total 400,000
D.Ratio of capital contributions less withdrawals made by the partners.
(RPCPA 1081,0586) The partners agreed to dissolve the partnership after selling the other asset for P200,000.
193. K, L, and M are partners with average capital balance during 2011 of P472,500, P238,650, Upon dissolution of the partnership, Axel should have received
and P162,350, respectively. The partners receive 10% interest on their average capital
balances; after deducting salaries of P122,325 to K and P82,625 to M, the residual profits or A. 0
loss is divided equally. B. 40,000
C. 60,000
In 2011, the partnership had a net loss of P125,624 before the interest and salaries to D. 70,000
partners. (Punzalan, 2015)
196. The following blance sheet is presented for the partnership of A, B, and C, who share
By what amount should K’s and M’s capital account change? profits and losses in the respectively ratio of 5:3:2.
K’s Capital Account M’s Capital Account Assets Liabilities and Capital
A. P40,844 decrease P31,235 decrease Cash P 120,000 Liabilities P280,000
B. P28,358 increase P32,458 increase Other Assets 1,080,000 A, Capital 560,000
C. P29,476 increase P17,536 increase B, Capital 320,000
D. P30,267 increase P40,448 decrease C, Capital 40,000
(Guerrero, 2013) Total P1,200,000 Total P1,200,000
194. Prior to partnership liquidation, a schedule of possible losses is frequently prepared to
determine the amount of cash that may be safely distributed to the partners. The schedule of Assume that the three partners decided to liquidate the partnership. If the other assets are
possible losses sold for P800,000, how should the available cash be distributed to each partner?
A B C
A. Consists of each partner’s capital account plus loan balance, divided by that A. 280,000 320,000 40,000
partner’s profit-and-loss sharing ratio. B. 324,000 236,000 16,000

C. 410,000 230,000 0 Accounts AB QR


D. 412,000 228,000 0 Cash 136,000 76,000
(Punzalan, 2015) Accounts Receivable 88,000 48,000
Inventories 304,000 364,000
197. Red, White, and Blue form a partnership on May 1, 2011. They agree that Red will Machinery 480,000 440,000
Accounts Payable 216,000 144,000
contribute office equipment with a toal fair value of P40,000; White will contribute delivery
Notes Payable 140,000 60,000
equipment with a fair value of P80,000; and Blue will contribute cash. If Blue wants a on third
interest in the capital and profits, he should contribute cash of:
A. P40,000 AB decided to pay off his notes payable from his personal assets. It was also agreed that QR
B. P120,000 inventories were overstated by P24,000 and AB machinery was over depreciated by P20,000. QR
C. P60,000 is to invest/withdraw cash in order to receive a capital credit that is 20% more than AB’s total net
investment in the partnership.
D. P180,000
(Guerrero, 2013) How much cash will be presented in the partnership’s statement of financial position?
A. 2,935,406.25
198. AK and BK decided to form a partnership on October 1, 2014. Their Statement of B. 2,218,125.00
Financial Position on this date were: C. 1,975,312.50
AK Bk D. 1,258,031.25
Cash 65,625.00 164,062.50 CPAR Pre-Boards October 2017
Accounts Receivable 1,487,500.00 896,875.00
Merchandise Inventory 875,000.00 885,937.50
Equipment 656,250.00 1,268,750.00
199. On December 1, 2014, MV and CD agreed to invest equal amounts and share profits
Total 3,084,375.00 3,215,625.00
equally to form a partnership. MV invested P3,120,000 cash and a piece of equipment. CD
Accounts Payable 459,375.00 1,159,375.00 invested some assets which are shown on the next page:
AK, Capital 2,625,000.00 Book value
BK, Capital 2,056,250.00 Accounts Receivable 400,000
Total 3,084,375.00 3,215,625.00 Inventory 1,120,000
Machineries, net 2,240,000
They agreed the following adjustments shall be made: Intangibles, net 920,000

• Equipment of AK is underdepreciated by P87,500 and that BK is overdepreciated by


The assets invested by CD are not properly valued, P32,000 of the accounts receivable are
P131,250.
proven uncollectible. Inventories are to be written down to P1,040,000. Included in the
• Allowance for doubtful accounts is to be set up amounting to P297,500 for AK and P196,875
machineries is an obsolete apparatus acquired for P384,000 with an accumulated depreciation
for BK.
balance of P336,000. Part of the intangibles is a patent with a carrying value of P56,000 which
• Inventories of P21,875 and P15,312.50 are worthless in the books of AK and BK
was sued upon by a competitor. CD unsuccessfully defended the case and the final decision of
respectively.
the court was released on November 29, 2014.
• The partnership agreement provides for a profit and loss ratio of 70% to AK and 30% to BK.
Assuming the use of transfer of capital method, how much is the agreed capital of AK to bring A. 274,000
the capital balances proportionate to their profit and loss ratio. B. 212,000
C. 486,000
On January 1, 2014, AB and QR agreed to form a partnership. The following are their assets and D. 374,000
liabilities:
CPAR Pre-Boards October 2017 202. CC Partnership began operations on June 1, 2014. On that date, CY and CR have capital
credits of P175,000 and P240,000, respectively. The partnership has the following profit-
sharing plan:
200. On December 1, 2014, MG and AN are combining their separate businesses to form a
a) 10% interest on partners’ capital balances at the end of the year
partnership. Cash and noncash assets are to be contributed. The noncash assets to be
b) P60,000 and P75,000 annual salaries for CY and CR, respectively.
contributed and the liabilities to be assumed are as follows:
c) Remaining profit will be divided to CY and CR on a 3:2 ratio,
MG AN respectively.
Book value Fair value Book value Fair value
Accounts Receivable 250,000 262,500 200,000 195,000 During the year, CY invested P150,000 worth of merchandise and withdrew P40,000 cash, while
Inventory 400,000 450,000 200,000 207,500 CR invested P120,000 cash. The partnership earned a profit of P266,375 during the year.
PPE 1,000,000 912,500 862,500 822,500
How much is CY’s capital balance at the end of 2014?
Accounts Payable 150,000 150,000 112,500 112,500
A. 266,375
MG and AN are to invest equal amount of cash such that the contribution of MG would be 10% B. 426,625
more than the investment of AN. C. 285,000
What is the amount of cash presented on the partnership’s statement of Financial Position on D. 150,000
December 1, 2014?
CPAR Pre-Boards October 2017
A. 1,344,000
B. 1,244,000 203. Cherryhill and Hace had been partners for several years, and they decided to admit Quincy
C. 3,120,000 to the partnership. The accountant for the partnership believed that the dissolved partnership
D. 2,180,000 and the newly formed partnership were two separate entities. What method would the
accountant have used for recording the admission of Quincy to the partnership?
CPAR Pre-Boards October 2017
A) the bonus method.
201. CC Partnership began operations on June 1, 2014. On that date, CY and CR have capital B) the equity method.
credits of P175,000 and P240,000, respectively. The partnership has the following profit- C) the goodwill method.
sharing plan: D) the proportionate method.
E) the cost method.
a.) 10% interest on partners’ capital balances at the end of the year
CPAR Pre-Boards October 2017
b.) P60,000 and P75,000 annual salaries for CY and CR, respectively.
c.) Remaining profit will be divided to CY and CR on a 3:2 ratio, respectively.

During the year, CY invested P150,000 worth of merchandise and withdrew P40,000 cash, 204. When the hybrid method is used to record the withdrawal of a partner, the partnership
while CR invested P120,000 cash. The partnership earned a profit of P266,375 during the year.
A) revalues assets and liabilities and records goodwill to the continuing partner but not to the
How much is CY’s capital balance at the end of 2014? withdrawing partner.
B) revalues liabilities but not assets, and no goodwill is recorded.
A. 5,025,000
C) can recognize goodwill but does not revalue assets and liabilities.
B. 2,512,000 D) revalues assets but not liabilities, and records goodwill to the continuing partner but not
C. 3,215,000 to the withdrawing partner.
D. 1,223,750 E) revalues assets and liabilities but does not record goodwill.
CPAR Pre-Boards October 2017 CPAR Pre-Boards October 2017
205. The disadvantages of the partnership form of business organization, compared to

corporations, include D. In accordance with their capital contributions


A) the legal requirements for formation.
(Millan, 2016)
B) unlimited liability for the partners.
C) the requirement for the partnership to pay income taxes. 210. When property other than cash is invested in a partnership, at what amount should the
D) the extent of governmental regulation.
noncash property be credited to the contributing partner’s capital account?
E) the complexity of operations.
CPAR Pre-Boards October 2017
206. The advantages of the partnership form of business organization, compared to A. Fair value at the date of contribution
corporations, include B. Contributing partner’s original cost
A) single taxation. C. Assessed valuation for property tax purposes
B) ease of raising capital. D. Contributing partner’s tax basis
C) mutual agency.
D) Limited liability. (Millan, 2016)
E) difficulty of formation. 211. The admission of a new partner effected through purchase of interest in the partnership is
CPAR Pre-Boards October 2017
207. The dissolution of a partnership occurs
A. Recorded in the partnership books as a debit to cash or other asset and credit
A) only when the partnership sells its assets and permanently closes its books. to the incoming partner’s capital account
B) only when a partner leaves the partnership.
C) at the end of each year, when income is allocated to the partners. B. Recorded in the partnership books as a transfer within equity
D) only when a new partner is admitted to the partnership. C. Recorded in the partnership books as a transfer from equity to liability
E) when there is any change in the individuals who make up the partnership.
CPAR Pre-Boards October 2017 D. Not recorded in its entirety
(Millan, 2016)

208. If a partner’s capital balance is credited for an amount greater than or less than the fair
212. State the proper order of liquidation
value of his net contribution, the excess or deficiency is called a
I. Outside creditors
A. Bonus
II. Owners’ interests
B. Goodwill
III. Inside creditors
C. Discount
A. I, III, II
D. Premium
B. I, II, III
(Millan, 2016)
C. III, II, I
209. If the partnership agreement does not specify how income is to be allocated, profits and
D. II, I, III
loss should be allocated
A. Equally (Millan, 2016)
B. In proportion to the weighted average of capital invested during the period
213. Lancelot is trying to decide whether to accept a salary of P40,000 or a salary of P25,000
C. Equitably so that partners are compensated for the time and effort expended
plus a bonus of 10% of net income after salary and bonus as a means of allocating profit among
on behalf of the partnership
the partners. Salaries traceable to the other partners are estimated to be P100,000. What amount capital accounts before recognition of partnership goodwill prior to Smith’s withdrawal was
of income would be necessary so that Lancelot would consider the choices to be equal? P252,000. After his withdrawal the remaining partners’ capital accounts, excluding their share
A. P 165,000 of goodwill, totaled P192,000. The total goodwill of the firm was:
B. P 290,000 A. P 144,000
C. P 265,000 B. P 168,000
D. P 305,000 C. P 192,000
(Dayag, 2015) D. P 300,000
214. MM, NN, OO are partners with capital balances on December 31, 2015 of P 300,000, P
(Dayag, 2015)
300,000 and P 200,000, respectively. Profits are shared equally. OO wishes to withdraw and it
is agreed that OO is to take certain equipment with second-hand value of P 50,000 and a note 217. The following condensed balance sheet is presented for the partnership of AA, BB, and

for the balance of OO’s interest. The equipment are carried on the books at P65,000. Brand CC, who share profits and losses in the ratio of 4:3:3, respectively:

new equipment may cost P 80,000. Compute for: (1) OO’s acquisition of the second-hand Cash P 160,000
equipment that will result to reduction in capital; (2) the value of the note that will OO get Other assets 320,000
from the partnership’s liquidation. Total P 480,000
A. (1) P15,000 each for MM and NN, (2) P150,000
B. (1) P5,000 each for MM, NN and OO, (2) P145,000 Liabilities P 180,000
C. (1) 5,000 each for MM, NN and OO, (2) P195,000 AA, Capital 48,000
D. (1) P7,500 each for MM and NN, (2) P145,000 BB, Capital 216,000
(Dayag, 2015) CC, Capital 36,000
215. RR and XX formed a partnership and agreed to divide initial capital equally, even though Total P 480,000
RR contributed P25,000 and XX contributed P21,000 in identifiable assets. Under the bonus The partners agreed to dissolve the partnership after selling the other assets for P200,000.
approach to adjust the capital accounts. XX’s unidentifiable assets should be debited for: Upon dissolution of the partnership, AA should have received
A. P 11,500
A. P 0
B. P 4,000
B. P 48,000
C. P 2,000
C. P 72,000
D. P 0
D. P 84,000
(Dayag, 2015)
(Dayag, 2015)
216. A. Smith, a partner in an accounting firm, decided to withdraw from the partnership,
218. When property other than cash is invested in a partnership, at what amount should the
Smith’s share of the partnership profits and losses was 20%. Upon withdrawing from the
noncash property be credited to the contributing partner’s capital account?
partnership he was paid P88,800 in final settlement for his interest. The total of the partner’s

A. Fair Value at the date of recognition C. Equitably so that partners are compensated for the time and effort expended
B. Contributing partner’s original cost on behalf of the partnership
C. Assessed valuation for property tax purposes D. In accordance with an established ratio
D. Contributing partner’s tax basis (Gleim)
(AICPA 0594 F-35)

223. On June 30, 2016, a partnership was formed by Mendoza and Lopez. Mendoza
219. A partnership records a partner’s investment of assets in the business at contributed cash. Lopez, previously a sole proprietor contributed non-cash assets
including a realty subject to a mortgage which was assumed by the partnership. Lopez’s
A. The market value of the assets invested capital account at June 30,2016 should be recorded at
B. A special value set by the partners
C. The partner’s book value of the assets invested A. The fair value of the property on June 30, 2016
D. Any of the above, depending upon the partnership agreement B. Lopez’s carrying amount of the property on June 30, 2016
(RPCPA 0598) C. The fair value of the property on June 30, 2016 less the mortgage payable
D. Lopez’s carrying amount of the property on June 30, 2016 less the mortgage
payable
220. In a partnership liquidation, the final cash distribution to the partners should be made in (BAYSA & LUPISAN, 2016)
accordance with the

A. Partners’ profit and loss sharing ratio For numbers 224 to 225 refer to the problem below:
B. Balances of the partners’ capital accounts
Diaz and Esteban entered into a partnership on February 1, 2016 by investing the following
C. Ratio of capital contributions made by the partners
assets:
D. Ratio of capital contributions less withdrawals made by the partners
(RPCPA 1079)

Diaz Esteban
221. As a result of the retirement of a partner in an existing partnership, the capital balance of Cash P 15,000
the remaining partners increases. If the assets of the partnership before retirement are
Merchandise Inventory P 45,000
properly valued, which of the following statements is true?
Land 15,000
A. The retiring partner received less than his capital balance before retirement
Building 65,000
B. There is partnership net loss prior to the retirement of the said partner
C. The remaining partner gives bonus to the retiring partner Furniture and Fixtures 100,000
D. There is impairment of existing assets recognized prior to retirement
(CPAR PREBOARD WEEK, 2017)
The agreement between Diaz and Esteban provides that profits and losses are to be divided into
222. If the partnership agreement does not specify how income is to be allocated, profits
40% and 60% to Diaz and Esteban respectively. The partnership is to assume the P30,000
should be allocated
mortgage loan on the building.
A. Equally
B. In proportion to the weighted-average of capital invested during the period
224. If Esteban is to receive a capital credit equal to his profit and loss ratio, how much cash (BAYSA & LUPISAN, 2016)
must he invest?

A. 77,500 228. Profit is the difference between


B. 97,500 A. assets and liabilities
C. 127500
B. the incoming cash and outgoing cash
D. 172,500
(BAYSA & LUPISAN, 2016) C. the assets purchased with cash contributed by the owner and the cash spent to operate
the business
D. the assets received for goods and services and the amounts used to provide the goods
225. Assuming Esteban invests P50,000 cash and each partner is to be credited for the full
amount of the net assets invested, the total capital of the partnership is and services

(Warren 9th ed.)


A. 210,000
B. 250,000 229. Which of the items below is not a business organization form?
C. 260,000
D. 290,000 A. Entrepreneurship C. Partnership
(BAYSA & LUPISAN, 2016)
B. Proprietorship D. Corporation
(Warren 9th ed.)
226. Assuming the partnership agreement provides that the partners should initially have an
230. An entity that is organized in which ownership is divided into shares of stock is a
equal interest in the partnership capital, what is Esteban’s capital upon partnership
formation? A. Proprietorship C. Partnership
A. 95,000
B. Corporation D. Governmental Unit
B. 105,000
C. 115,000
D. 125,000
(BAYSA & LUPISAN, 2016) (Warren 9th ed.)

231. Financial reports are used by


227. Canlas, a partner in the 3C Partnership, has a 30% participation in partnership profits and A. Management C. Inventors
lossess. Canlas’ capital account had a net decrease of P120,000 during the calendar year
2016. During 2016, Canlas withdrew P260,000 (charged against his capital account) and B. Creditors D. All are correct
contributed property valued at P50,000 to the partnership. What was the profit of 3C (Warren 9th ed.)
Partnership?
232. Which of the following is not a characteristic of a corporation?
A. 300,000 A. Corporations are organized as a separate legal taxable entity
B. 466,667 B. Ownership is divided into shares of stock.
C. 700,000
D. 1,100,000

C. Corporations experience an ease in obtaining large amounts of resources by issuing A, B and C decided to form ABC Partnership. It was agreed that A will contribute an equipment
stock. with assessed value of P100,000 with historical cost of P800,000 and accumulated depreciation of
D. A corporation’s resources are limited to their individual owners’ resources. P600,000. A day after the partnership formation, the equipment was sold for P300,000.

(Warren 9th ed.)


B will contribute a land and building with carrying amount of P1,200,000 and fair value of
233. He refers to a partner who contributed not only money and property but also P1,500,000. The land and building are subject to a mortgage payable amounting to P300,000 to be
industry to the newly formed partnership. assumed by the partnership. The partners agreed that B will have 60% capital interest in the
partnership. The partners also agreed that C will contribute sufficient cash to the partnership.
A. industrial partner
B. nominal partner 236. What is the total agreed capitalization of ABC Partnership?
C. capitalist-industrial partner A. 1,500,000
D. capitalist partner B. 2,000,000
(CPAR handout, 2018) C. 2,500,000
D. 3,000,000
234. It refers to a type of partnership wherein all partners are liable to the creditors pro-
rata up to the extent of personal or separate assets after the partnership’s asset are 237. What is the cash to be contributed by C in the ABC Partnership?
exhausted. A. 500,000
B. 600,000
A. General partnership C. 700,000
B. Partnership by estoppel D. 800,000
C. Limited partnership (CPAR handout, 2018)
D. Particular partnership
(CPAR handout, 2018)
238. When property other than cash is invested in a partnership, at what amount should
235. Which of the following will decrease the capital balance of a partner? the noncash property be credited to the contributing partner’s capital account?
A. Share in partnership profit
A. Contributing partner’s tax basis
B. Receipt of share in revaluation surplus from a partnership property, plant and
B. Contributing partner’s original cost
equipment C. Fair value at the date of contribution
C. Drawing made by partner D. Assessed valuation for property tax purposes
(Wiley 2014)
D. Advances made by a partner to the partnership
239. In a limited partnership, a general partner
(CPAR handout, 2018) A. Is excluded from management
B. Is not entitled to a bonus at the end of the year
C. Has limited liability for partnership debt 245. JJ and KK are partners who share profits and losses in the ratio of 60% and 40%
D. Has unlimited liability for partnership debt respectively. JJ’s salary is P60,000 and P30,000 for KK, the partners are also paid interest
(BCA & L 10e) on their average capital balances. In 2012, JJ received P30,000 of interest and KK, P12,000.
240. Partnership drawings are The profit and loss allocation is determined after deductions for the salary and interest
A. Usually maintained in a separate account from the partner’s capital account payments. If KK’s share in the residual income (income after deducting salaries and
B. Equal to partner’s salaries interest) was P60,000 in 2012, what was the total partnership income?
C. Similar to advances made to partners and are included as assets on the balance sheet
D. Not discussed in the specific contract provisions of the partnership A. P192,000
(FT & C 11e) B. P345,000
241. Which of the following is an advantage of a partnership? C. P282,000
A. Mutual agency D. P387,000
B. Limited life (Dayag 11)
C. Unlimited liability
D. None of these
(J & C 3e) FINISH!
242. The profit and loss sharing ratio should be A year ago, you did not know today.
A. In the same ratio as the percentage interest owned by each partner
You did not know how you’d make it here.
B. Based on relative effort contributed to the firm by the partners
C. A weighted average of capital and effort contributions But you made it here!
D. Based on any formula that the partners choose
By grace, you made it here! 😊
(J & C 3e)
243. Maxwell is trying to decide whether to accept a salary of $60,000 or a salary of - (M.H.N)
$25,000 plus a bonus 0f 20% of net income after the bonus as a means of allocating profit
among the partners. What amount of income would be necessary so that Maxwell would
consider the choices to be equal?
A. $35,000
B. $85,000
C. $140,000
D. $210,000
(FT & C 11e)
244. Joan a senior partner in a fashion designing firm has a share of 30% in earnings. In
19x8, she transferred to the firm, property with current fair value of P25,000 but made
capital withdrawal of P130,000. If her closing capital balance was P60,000 less than her
beginning capital balance, how much was the partnership’s net income loss in 19x8?

A. P(45,000)
B. P135,000
C. P150,000
D. P180,000
(RRCPA 0598)

Potrebbero piacerti anche